41
SECTION 1 READING COMPREHENSION Passage 1: Mexican-American Literature Our humanities passage is relatively straightforward, even if the author’s language is occasionally convoluted. As usual, the Topic (Mexican-American literature), is clear within the first few lines. The Scope becomes apparent somewhat later—the author is especially interested in Mexican-American literature as an expression of cultural transition. The Purpose and Main Idea are not obvious at this point, so we forge ahead. Paragraph 1: The author focuses our attention on the way that literature written by Mexican-Americans reflects a mix of the two cultures, in the use of both English and Spanish. Further blending of the cultures is seen in the writers’ “Mexican sensibility” and use of American settings. The author distinguishes Mexican-American literature from Mexican literature, as reflecting differing content and concerns. In paragraph 2, the author explains why and how Mexican American writers are influenced by the mostly agrarian culture of their ancestors. Paragraph 3 expands on the author’s contrast between Mexican literature and Mexican-American literature. In Mexico, literature is dominated by the “prominent literary establishment.” This Mexican literary establishment views Mexican American literature as merely “regional” writing. The author’s point of view begins to reveal itself at the very end of this paragraph: the “apparent simplicity” of Mexican American literature disguises its “thematic richness.” The author’s view is further developed in paragraph 4, which returns to the first paragraph’s notion of cultural blending, with a twist. Mexican American writers don’t seek assimilation into American culture. Rather, through their writings they create a new identity that contains both Mexican and American elements. At this point, we see the author’s Purpose and Main Idea, to show that Mexican- American literature reflects the cultural transition of Mexican immigrants to the U.S., as they create a new identity combining elements of both cultures. Our Roadmap looks something like this: ¶1: Mex-Am literature: often mix English and Spanish; explores Mexican content and concerns in U.S. setting ¶2: M-A. lit. reflects agrarian culture ¶3: Mex. Lit.: dominated by literary establishment. Cf. M-A lit—seems simple; thematically rich ¶4: M-A writing shows cultural transition, new identity reflects both Mexican and American social realities 1. (C) Global (Main Point) Where a passage focuses on a comparison, the main point is generally a value judgment about one or both things that are being compared. The author spends much time comparing Mexican American literature to Mexican literature. While the Mexican literary establishment dismisses Mexican American literature as merely “regional,” the author sees this literature as depicting the creation of a new identity for Mexican Americans, combining elements of both Mexican and American cultures. That’s answer choice (C). (A) is half right, half wrong. The literature does reflect a cultural transition, but the author is not primarily concerned with the cause of the transition. (B) Here we have a faulty use of detail. This answer choice focuses on details from paragraphs 2 and 3, but not the main point of the passage as a whole. (D) Distortions all the way. The author notes that the content and concerns of Mexican American literature differs from that of Mexican literature, but does not call it “unique.” There is no mention of “stylistic innovation” of Mexican American writers. The author talks about how the Mexican literary establishment controls the reputation and success of Mexican writers, but does not discuss Mexican American literature in terms of reputation and prominence. (E) Another faulty use of detail. Paragraph 2 discussed the relationship between Mexican Americans and their agrarian ancestors, but that’s not the main point of the passage as a whole. 2. (D) Inference The right answer choice for an Inference question must be true, based solely upon the text. We are given no clues as to what specifically the testmakers would have us infer, so we evaluate the answer choices in turn. If time were running short for you, this might have been a good place to guess or skip.

LSAT_PT 50_Expl_web

Embed Size (px)

Citation preview

Page 1: LSAT_PT 50_Expl_web

SECTION 1 READING COMPREHENSION Passage 1: Mexican-American Literature Our humanities passage is relatively straightforward, even if the author’s language is occasionally convoluted. As usual, the Topic (Mexican-American literature), is clear within the first few lines. The Scope becomes apparent somewhat later—the author is especially interested in Mexican-American literature as an expression of cultural transition. The Purpose and Main Idea are not obvious at this point, so we forge ahead. Paragraph 1: The author focuses our attention on the way that literature written by Mexican-Americans reflects a mix of the two cultures, in the use of both English and Spanish. Further blending of the cultures is seen in the writers’ “Mexican sensibility” and use of American settings. The author distinguishes Mexican-American literature from Mexican literature, as reflecting differing content and concerns. In paragraph 2, the author explains why and how Mexican American writers are influenced by the mostly agrarian culture of their ancestors. Paragraph 3 expands on the author’s contrast between Mexican literature and Mexican-American literature. In Mexico, literature is dominated by the “prominent literary establishment.” This Mexican literary establishment views Mexican American literature as merely “regional” writing. The author’s point of view begins to reveal itself at the very end of this paragraph: the “apparent simplicity” of Mexican American literature disguises its “thematic richness.” The author’s view is further developed in paragraph 4, which returns to the first paragraph’s notion of cultural blending, with a twist. Mexican American writers don’t seek assimilation into American culture. Rather, through their writings they create a new identity that contains both Mexican and American elements. At this point, we see the author’s Purpose and Main Idea, to show that Mexican-American literature reflects the cultural transition of Mexican immigrants to the U.S., as they create a new identity combining elements of both cultures. Our Roadmap looks something like this: ¶1: Mex-Am literature: often mix English and Spanish; explores Mexican content and concerns in U.S. setting ¶2: M-A. lit. reflects agrarian culture

¶3: Mex. Lit.: dominated by literary establishment. Cf. M-A lit—seems simple; thematically rich ¶4: M-A writing shows cultural transition, new identity reflects both Mexican and American social realities 1. (C) Global (Main Point) Where a passage focuses on a comparison, the main point is generally a value judgment about one or both things that are being compared. The author spends much time comparing Mexican American literature to Mexican literature. While the Mexican literary establishment dismisses Mexican American literature as merely “regional,” the author sees this literature as depicting the creation of a new identity for Mexican Americans, combining elements of both Mexican and American cultures. That’s answer choice (C). (A) is half right, half wrong. The literature does reflect a cultural transition, but the author is not primarily concerned with the cause of the transition. (B) Here we have a faulty use of detail. This answer choice focuses on details from paragraphs 2 and 3, but not the main point of the passage as a whole. (D) Distortions all the way. The author notes that the content and concerns of Mexican American literature differs from that of Mexican literature, but does not call it “unique.” There is no mention of “stylistic innovation” of Mexican American writers. The author talks about how the Mexican literary establishment controls the reputation and success of Mexican writers, but does not discuss Mexican American literature in terms of reputation and prominence. (E) Another faulty use of detail. Paragraph 2 discussed the relationship between Mexican Americans and their agrarian ancestors, but that’s not the main point of the passage as a whole. 2. (D) Inference The right answer choice for an Inference question must be true, based solely upon the text. We are given no clues as to what specifically the testmakers would have us infer, so we evaluate the answer choices in turn. If time were running short for you, this might have been a good place to guess or skip.

Page 2: LSAT_PT 50_Expl_web

(A) is a 180. The author sees the Mexican literary establishment as foolishly dismissing Mexican American literature as “regional” due to its apparent simplicity, while missing the thematic complexity of the writing. Why should Mexican American writers want to create a comparable establishment in the United States? (B) While the use of both English and Spanish in Mexican American literature is noted by the author, this was not portrayed as evidence of a “brief transitional period.” (C) This one’s extreme. The Mexican literary establishment is not impressed by Mexican American writers, but the author never suggests that the writers offended or caricatured anyone. (D) The author is clearly most impressed by the thematic content of Mexican American literature, and describes its “apparent simplicity.” This one is a winner—it’s certainly in line with the author’s focus. For the record: (E) Another 180. The author never suggests that Mexican American writers are losing their connection to Mexico. 3. (C) Inference Look for Hot Words that can help you predict the answer to an Inference question. “Many Mexican American writers” is a phrase we see in line 47. At that point in the passage, the author notes that these writers assert that they are not trying to assimilate into American society. That’s (C). (A), (B), (D) and (E) are outside the scope of the passage. The author never compares Mexican American writing to Mexican writing in terms of stylistic innovation. The author never indicates how Mexican American writers feel about any literary establishment, whether Mexican or American. The author never suggests that Mexican American writing lacks in critical acceptance due to its authors’ use of both English and Spanish. The author never compares Mexican American literature to American literature. 4. (B) Detail A good understanding of the scope of the passage helps you avoid wrong answer choices on questions that that ask you to spot one passage detail from the five answer choices.

Here we have another question that forces us to consider each answer choice in turn, without offering us an opportunity to predict the right answer. Because we are looking for something specifically mentioned in the passage, our understanding of the scope of the passage and our Roadmap will be very helpful in getting us to the right answer choice quickly. Wrong answer choices will be outside the scope of the passage. (A) The author never mentions any specific work of Mexican American literature. Bad answer. (B) Paragraph 2 is all about the reasons why much Mexican American literature reflects agrarian themes or topics. This is our good answer. For the record: (C) The passage notes that the Mexican literary establishment dismisses Mexican American literature as merely “regional,” but makes no mention of how American critics view the literature. (D) The passage does not discuss any possible influence U.S. literature may have had on Mexican American writers. (E) While the author tells us that Mexican American writers tend to write in both English and Spanish, we are not told which language predominates. 5. (D) Inference One way the LSAT writers can make a passage more difficult is by providing few research clues in the questions. Yet another question with no clues. Waste no time before you evaluate the answer choices. (A) The author notes that Mexican American writing frequently reflects agrarian themes and topics, but never suggests the agrarian way of life is portrayed as a cure for the alienation of modern culture. (B) There is no discussion of immigrant literature in general. (C) While the author tells us that much of Mexican writing has been criticized for being dominated by the Mexican literary establishment, the author never suggests that Mexican American literature is more experimental than Mexican literature due to any lack of domination by a comparable establishment. (D) In lines 36-40, the author notes that the Mexican literary establishment views Mexican American

Page 3: LSAT_PT 50_Expl_web

writing as “regional,” suggesting that such writing is not valued by the Mexican literary establishment. Earlier in that same paragraph, the author notes that Mexican writing tends to have an urban focus. These two pieces of evidence support answer choice (D) as a good inference. (E) The author never compares the attachment to history of Mexican American writers versus Mexican writers. Passage 2: Bankruptcy laws With a clear author’s voice and 8 questions, including 3 Global questions, this passage was a good candidate to attack first. The passage announces its Topic (bankruptcy laws), Scope (their shift from bankruptcy as a punishment to a remedy for debtors and creditors), Purpose (to argue that the modern, remedial approach is better), and Main Idea (returning to a more punitive approach is a bad idea) in the first paragraph. These ideas are further developed in the remainder of the paragraphs. The second paragraph explains how the earlier punitive approach to bankruptcy did not serve creditors or society. The third paragraph describes how modern bankruptcy laws allow debtor corporations or individuals to contribute to society by continuing to operate or earn income. These laws not only are good for society, but they are also good for creditors. Our Roadmap might look something like this: ¶1: BK laws: Past--punish debtors. Present—remedy for those in financial trouble. Punishment not good for society ¶2: punitive approach didn’t serve creditors or society ¶3: Modern BK laws serve public good, best for creditors 6. (A) Global—Main Point Sometimes the author explicitly states the main point of the passage. The author is not shy about stating his position: “the temptation to return to a focus on punishment of individuals or corporations that become insolvent must be resisted.” (lines 14-16) (A) captures that idea nicely.

(B) distorts the author’s point. Modern bankruptcy laws better serve creditors than the old punitive approach, but the shift was driven by a realization that the public good was better served by allowing financially distressed corporations and individuals to continue their economic activity. (C) is a 180. According to the author, the punitive approach was not more economically efficient than the modern approach. (D) The author never discusses the deterrent value of a punitive approach to bankruptcy. (E) is another distortion. The author notes that modern bankruptcy laws better serve the needs of an interdependent society, but that interdependence is never described as the impetus for the shift in focus from punishment to remediation. 7. (D) Inference Not every question with a line reference is a detail question. Here, we are asked what the author implies by using specific language at a specific point in the passage. Putting the line reference mentioned in the question into context, we see that the shift in focus of the bankruptcy laws was intended to help individuals and corporations in financial distress. When the author says that this “unexpectedly” provided a remedy for creditors, he suggests that although the laws weren’t necessarily designed to help creditors, they did. That’s what answer choice (D) says. (A) and (B) are extreme. While creditors may have been unexpected beneficiaries of bankruptcy reform, “surprise” (A) is too strong a word to describe the situation, and the author never suggests that the creditors are the “chief” (B) beneficiaries of modern bankruptcy laws. The passage does not describe the formulation of modern bankruptcy laws, and so (C) is outside the scope. (E) is a distortion we already dismissed in question 6. While the author notes that we live in an “interdependent society,” and that modern bankruptcy laws better serve the needs of creditors and debtors as well as society at large, there is no suggestion that this interdependence “triggered” the formation of modern bankruptcy laws. 8. (A) Author’s Attitude (Global)

Page 4: LSAT_PT 50_Expl_web

Characterizing the author’s attitude as positive, negative, or neutral, can make short work of an author’s attitude question. The author clearly favors the evolution of bankruptcy law toward remediation of the debtor’s financial distress, and away from a punitive approach. Recognizing the author’s positive attitude allows us to quickly eliminate answer choices (C), (D), and (E), as the author was neither neutral (C), skeptical (D), nor concerned (E) about the way bankruptcy laws have evolved. (A) perfectly captures the author’s approval of the modern, remedial approach, and disapproval of the old punitive approach, under which debtors were denied the opportunity to work and pay off all or at least part of their debt. Since the author never discusses possible changes to the current law, (B) is outside the scope of the passage, and the fourth wrong answer choice. 9. (E) Global—Primary Purpose Critical reading means that you have a good understanding of the author’s purpose before you answer any questions about the passage. The passage presents the author’s argument against reverting back to a punitive approach toward people and businesses that are in financial distress. That’s what (E) says. (A) is half right, half wrong. The author criticizes past approaches, but praises present approaches as better meeting the needs of debtors, creditors, and society overall. The author does not compare the practices of bankruptcy courts past and present, and so (B) is outside the scope. (C) is extreme. The author clearly would argue against reverting to a more punitive approach to bankruptcy, but does not express an opinion as to whether other changes might be appropriate. Likewise, (D) is outside the scope, because the author does not suggest ways in which current bankruptcy laws might be improved. 10. (A) Inference A good Roadmap can make researching an inference question much easier.

As noted in our Roadmap, the author discusses the punitive theory of bankruptcy legislation in paragraph 2. Lines 21-29 describe how an insolvent debtor was seen as being either unwilling to pay his debts or else negligent. The debtor having broken “sacrosanct social contracts,” it was generally believed that imprisonment of such people was necessary to remove such people from society. That sounds a lot like (A). (B) is a distortion—the recognition that insolvency harms society as a whole serves as the basis for the modern approach, not the old punitive approach. Insolvency that might be justified given the potential gains of the risk-taking debtor is beyond the scope of this passage, and not at all part of the mindset of those advocating a punitive approach, so (C) is out. (D) is a 180. The old approach was to dissolve businesses that were insolvent, as part of the punishment. (E) is extreme—while the author noted that employees of an insolvent business could be hurt by the dissolution of the business under the old approach, there is no suggestion that blame extended to the employees of a bankrupt business. 11. (B) Inference (with a Global flavor) Kaplan strategies work even when the question is unusual. Our Roadmap and understanding of the scope, author’s purpose, and main idea will help us get to the right answer. With these in mind, we evaluate each answer choice in turn. (A) The author likes current bankruptcy laws and dislikes the old approach. This is a 180. (B) In the last paragraph, the author talks not only about how modern bankruptcy laws best serve the interests of society as a whole, but also about certain aspects of modern bankruptcy law that may be seen as somewhat punitive. That punitive aspect of the modern approach, is secondary to the goal of restoring debtors to financial health. Answer choice (B) best captures this acknowledgement that punishment may still play a small role in bankruptcy laws. For the record: (C) The author says nothing about criminal law—this answer choice is outside the scope of the passage.

Page 5: LSAT_PT 50_Expl_web

(D) is likewise outside the scope of the passage. The author makes no suggestions about future changes to bankruptcy law. (E) contradicts the author’s point about the old approach to bankruptcy. While the author certainly acknowledges the burden placed on debtors under the old approach, she never suggests that under the new approach, creditors bear the burden of insolvency. Rather, the new approach gives creditors their best chance at recovering the debt. 12. (D) Inference Eliminate answer choices that go beyond the scope of the passage. The author’s focus, when it comes to change in the bankruptcy laws, is on the change from a punitive approach to a more remedial approach. According to the author, this change best served society’s interests overall. That’s answer choice (D). (A) is extreme. The passage only discusses one change in bankruptcy laws, not all changes. (B) The author never says anything about whether courts or the legislature initiated the changes to bankruptcy laws. Outside the scope. (C) Another one outside the scope of the passage. The author never discusses creditors’ reactions to bankruptcy reform. (E) The author never discusses broader social trends beyond the changes to bankruptcy laws. Another answer choice that’s outside the scope of the passage. 13. (E) Logic (Weaken) Treat logic questions in reading comprehension the same way you do in the logical reasoning section. This question asks us to identify a reason why punishment might be a good approach to insolvency. Since the author’s argument rests on the contention that the remedial approach better meets the needs of society as a whole, her argument would be weakened by evidence that countries that retained a punitive approach do better economically than those who have abandoned the punitive approach. That’s (E) in a nutshell. (A) This one is tempting, but wrong because debtors’ improved behavior following imprisonment says nothing about whether imprisonment actually

benefits society as a whole. Just because someone acts more responsibly doesn’t mean they are henceforth immune from insolvency and its accompanying problems. (B), if anything, strengthens the author’s argument by providing an example of how bankruptcy of a large company can harm a community even where the company’s employees find work elsewhere. (C) The number of bankruptcy filings says nothing about the overall effect of bankruptcy on a community, nor does the number indicate whether a punitive approach is better than a remedial approach. (D) Evidence that a community was able to experience robust economic growth after the liquidation of a large insolvent company says nothing as to whether the community would or would not have been better served by a reorganization that would have allowed the company to stay in business. Passage 3: Forging a National Identity The author of this passage takes his time letting us know just what the passage is about. We are well into Paragraph 1 before we see that the Topic of the passage is national identity, and the Scope of the passage is how countries revise their national identity to reflect their current cultural values. The author begins his exploration of the topic by considering Greek national identity, noting that although Greek civilization has roots in Egypt, Africa, and Europe, in the 19th century, the Greeks revised their national identity to support an image of European cultural dominance. Paragraph 2 offers another example of how a country could use “manufactured” or “reinterpreted” traditions to establish national identity. Specifically, the author describes how in 1876, the English used “jamboree” celebrations to give a sense of history to Queen Victoria’s new position as Empress of India. Paragraph 3 turns our attention from colonizing nations to those who were colonized, noting that in Algeria, pre-colonial culture was idealized during that country’s “decolonization” from French occupation. This idealization of pre-colonial culture gave the newly independent Algerians a national identity they could revive and admire. The author ends the passage with a paradox. In Paragraph 4, the author notes that although all nationally defined cultures aspire to sovereignty and dominance, today (more than ever) we recognize

Page 6: LSAT_PT 50_Expl_web

that national identity is also influenced by many “foreign” elements. The author’s Main Idea is that although countries try to forge an independent national identity, national identity is the product of many historical and cultural experiences. The author’s Purpose is to describe some processes by which national identity develops. Our Road Map: ¶1: Nat’l identity can be revised to reflect cultural values. Ex: Greeks ¶2: “Tradition” can be used to help establish new national identity. Ex: Queen Victoria, Empress of India ¶3: Idealized, pre-colonial culture revived after decolonization. Ex: Algeria ¶4: Paradox: Nat’l identity goal is sovereignty and dominance, but now we see foreign elements in nat’l identity 14. (E) Global (Main Point) The correct answer choice for a Main Point question will justify the inclusion of just about everything in the passage. As we saw in our initial review of the passage, the passage not only describes the ways that countries try to establish their own national identities, but also how national identity is influenced by foreign elements as well. That’s answer choice (E). (A) is one huge distortion. According to the passage, fabricating traditions and rituals was one way national identity was established (remember Queen Victoria), but the author never suggested that imperial societies ignored the self-understanding of native cultures, and never described native cultures as heterogeneous. (B) is another distortion. Attempting to reconstruct pre-colonial societies, as well as fabricated traditions and rituals, are described by the author as ways that cultures establish national identity. The author never suggests they are essentially the same, and these two strategies are examples that support the author’s main idea about how societies form their national identities. (C) is a faulty use of detail. Both the Greeks and colonizing societies were used by the author as examples of societies that revised their history to create a national culture.

(D) is half right, half wrong. The author does note that most colonized societies have gained their independence, but never suggests that those societies must discard the trappings of imperial culture to regain their pre-colonial cultures. 15. (C) Detail EXCEPT If you are pressed for time, you may want to save a Detail EXCEPT question for the end of the question set. Your work on other questions will improve your understanding of the scope of the passage, and will help you eliminate wrong answer choices. No clues here to guide our research before we evaluate the answer choices, so we’ll just have to use hot words from the answer choices to guide our research. (A) In lines 6-9, the author describes the influences that affect national identities that people construct from their past. (B) In lines 17-21, the author explains why 19th century European commentators ignored ancient Greek writers’ acknowledgements of African and Egyptian influences on Greek culture. (C) The author never specifically discusses exactly how African cultural influences affected the culture of ancient Greece. We have a good answer choice. For the record: (D) In lines 27-39, the author explains why Queen Victoria was declared empress of India. (E) In lines 45-49, the author discusses a reason why revolutionary poets speak and write as they do. 16. (A) Author’s attitude (Inference) When an author provides examples that support a study, his attitude toward the study is positive. The first thing we do is go back to line 6 to identify which studies the question concerns. We see that the studies argue that present anxieties and agendas exert tremendous influence on the national identities we construct from our past. The bulk of the passage then sets out example after example of societies constructing and revising their national identities. (A) is the only answer choice that captures the author’s positive attitude toward the studies. (B) (reservation), (C) (skepticism), (D) (concern), and (E) (unqualified disagreement) are all negative,

Page 7: LSAT_PT 50_Expl_web

and do not describe the attitude the author has demonstrated in the passage. 17. (D) Logic Function Context is the key to determining the function of a detail. The author’s use of quotation marks around the word “traditional” in line 37, reflects an ironic use of the word. A jamboree celebration is observed after a lengthy period of time—in the example of Queen Victoria’s jamboree as empress of India, the celebration was an attempt to legitimize Queen Victoria’s power over India, as if she had been empress of India for years and years, as opposed to a reign based on a recent edict. (D) captures this usage of the term. (A) The author never suggests that Queen Victoria’s jamboree celebrations were a revival of a long-neglected tradition. (B) The passage never describes jamboree celebrations as part of native culture. (C) The jamboree celebrations were a way of giving an air of legitimacy to colonial rule, and did not exemplify the dominance of imperial culture. (E) There was nothing in the native culture that gave rise to the jamboree celebrations. 18. (C) Inference Take advantage of an author’s repetition and restatement of ideas in the passage. The question puts us squarely into paragraph 4, where the author contrasts “imperial attitudes of uniqueness and superiority” with our modern recognition of the commonalities between different cultures. The “purveyors of nationalist dogma” are clearly identified with those who gravitate toward a view of national identity that is “unitary, monolithic, or autonomous.” (C) is the only answer choice that captures that distinction. (A) goes beyond the scope of the passage. The two views of national identity described in paragraph 4 do not prescribe how countries should define their national identity. Rather, the competing views see national identity either as unique/monolithic, or partaking of foreign elements. (B) and (D) describe attitudes that would be associated with those who recognize the commonalities between cultures, and the way that

transnational cultural sharing is virtually unavoidable. (E) is another answer choice that is beyond the scope of the paragraph, and more properly fits in with the ideas expressed in the first three paragraphs of the passage. 19. (D) Logic (Parallel Reasoning) Parallel Reasoning questions in reading comprehension generally require comparison of an overall situation with a rule or another situation. This question acts like a parallel reasoning question where we must compare the stimulus as a whole (in this case, the specific line reference) to the answer choices to find the parallel situation. The “rituals, ceremonies, and traditions” mentioned in lines 26-27 of the passage were fabricated by colonizers to validate and support their dominant position over the colonized nation. (D) describes just such a ritual. (A), (B), (C), and (E) all describe rituals that are either actual rituals, traditions, and ceremonies from the native culture, or they honor native culture in some way, which is exactly the opposite intention of the rituals described in the question stem. 20. (A) Logic (Principle) The right answer choice for a principle question matches the situation described (or in this case, referred to) in the stimulus, but also applies to other similar situations. The examples in the second and third paragraph demonstrate ways that cultures revise their national identity to reflect current values. This revision is described in answer choice (A). (B) is outside the scope of the paragraphs referred to in the question stem, and distorts what the passage says about cultural uniformity. According to the author, “purveyors of nationalist dogma” may perceive their national identity as monolithic and autonomous, but that view of national identity is not grounded in reality. (C) is extreme. The author describes certain rituals as contrived or fabricated, but does not go so far as to characterize most colonial cultures as artificial and contrived. (D) is a faulty use of detail, from paragraph 4.

Page 8: LSAT_PT 50_Expl_web

(E) is a distortion. The author never compares the authenticity of revolutionary versus imperial cultures. 21. (D) Global Always check for a Global question at the end of the question set—you may be pleasantly surprised. Although the passage is full of dense and often abstract language, our Roadmap makes the author’s purpose clear. The author provides many examples to support his claim that national identity can be and has been revised to reflect current values and concerns. That’s answer choice (D). (A) goes far beyond the scope of the passage. The author is concerned with the way existing cultural identity is formed and partakes of multiple influences, not with arguing for the creation of a global culture. (B) distorts the author’s point. Imperialist attitudes include a desire for cultural uniformity, according to the passage. (C) The author implies that an objective view of cultural identity reveals foreign influences in virtually all cultures. That viewpoint supports the author’s main argument, that cultural identity reflects current needs and values. (E) is a faulty use of detail and a distortion, all rolled into one. 19th century scholars are mentioned in paragraph 1. The author notes that during that time, the Greeks were creating a national identity based on European cultural dominance, ignoring African, Egyptian, and Eastern influences on Greek culture. The scholars are described as having “passed over without comment” ancient Greek writers’ acknowledgement of the hybrid past of Greek culture. The motivations of the scholars are not discussed. Passage 4: Shaking the Foundation of Scientific Research. Although this natural science passage is broken up into five attractively short paragraphs, offers seven questions as a reward for getting through the passage, and begins with good contrast, test-takers not familiar with terms like “chaos theory” or “riddled basins of attraction” likely pegged this as a difficult passage best attacked toward the end of the section. In addition to the abstract and technical language, many of the questions have very long answer choices, which will be time-consuming to

read and evaluate even if the topic of the passage were more accessible. However, difficult passages such as this one will reward the student who stays calm and uses the Kaplan method exactly the same way she would on an easier passage. We begin by looking for the Topic, announced within the first few lines: scientific research. The Scope is apparent by the end of the first paragraph: Sommerer and Ott’s research of a physical system in which certain results cannot be replicated. We’ll need to keep reading to obtain a clear idea of the author’s Purpose and Main Idea. Paragraph 1 sets out the contrast: although a foundation of scientific research is that results are credible only if they can be replicated, Sommerer and Ott have identified a physical system where the motion of a particle cannot be replicated. Paragraph 2 describes the basis for the physical system—an analogy to riddled basins of attraction. The author then defines the term “basin of attraction.” Paragraph 3 is where the author explains the significance of a “riddled basin of attraction” – at the boundaries of such a basin of attraction, one cannot predict where spilled water will flow. In Paragraph 4, the author describes how Sommerer and Ott distinguish their system from chaos. In chaos, a particle’s general destination, but not its specific path, could be predicted. In a riddled basin of attraction, it is not possible to predict even the general destination of a particle. Finally, in Paragraph 5, the author explains the significance of Sommerer’s and Ott’s work. Because they came up with their model based on the first one they attempted, it is likely that other unstable systems exist. If that is true, then it is possible that some unrepeatable research results are not due to any flaw in the research. This would force scientists to question one of the foundations which guide their work. Now we can see the author’s Purpose and Main Idea. The author’s Purpose is to describe Sommerer’s and Ott’s model and the significance of that model. The author’s Main Idea is that Sommerer’s and Ott’s model may explain why certain research results could be both unrepeatable and scientifically valid, contrary to traditional principles that say that only research results that are replicable can be valid.

Page 9: LSAT_PT 50_Expl_web

Our Roadmap might look like this: ¶1: Sommerer and Ott: posit physical system where motion of particle can’t be replicated ¶2: Basis of system: analogy to riddled basins of attraction; basins of attraction defined ¶3: in riddled basins of attraction, can’t predict where water will go ¶4: S and O distinguish their system from chaos. ¶5: Probably other unstable systems exist; may explain failed attempts to replicate results 22. (B) Global (Main Point) Always read for the gist even if you don’t fully understand technical terms used in the passage. Answer choice (B) matches our articulation of the main idea of the passage, and confirms our understanding of the author’s big idea. Sommerer and Ott’s model explains why some research that has been dismissed due to an inability to replicate the results may nevertheless be scientifically valid. This contradicts what has been a fundamental tenet of scientific research. (A) distorts the author’s point. (C) is extreme. Sommerer and Ott did not suggest that no research can truly be replicated; only that some research that could not be replicated may in fact be valid. (D) is also extreme. Riddled basins of attraction are one example of an unstable physical system. The author never suggests that Sommerer and Ott’s model applies to a majority of physical systems studied by scientists. (E) This one’s a 180. Sommerer and Ott’s model suggests that some experimental results that cannot be replicated may nevertheless be valid. 23. (D) Logic Function Use your Roadmap to locate the context in which an author uses a particular detail. “Chaos” is mentioned only in paragraph 4. In that paragraph, the author notes that Sommerer and Ott’s model is even more uncertain than the uncertainty known as “chaos,” because at least in a chaotic system, a particle’s general destination would be known, even though its specific path and

general destination could not be determined. In Sommerer and Ott’s model, an observer could not even predict the general destination of a given particle. That extreme uncertainty is captured in answer choice (D). Neither the number of physical irregularities (A), nor unusual types of physical irregularities (B) within a riddled basin of attraction is even mentioned in paragraph 4. The passage never distinguishes riddled basins of attraction from other basins of attraction based on the percentage of the riddled basin that exhibits uncertainty, so (C) is a distortion. Likewise, the number of fractal properties (E) of a riddled basin of attraction create the complete lack of certainty found in the physicists’ systems, but the comparison to chaos focuses on the uncertainty created by the riddled basins of attraction, not the number of their fractal properties. 24. (C) Inference Your understanding of the scope and main idea of the passage can help you eliminate wrong answer choices on an inference question. The author is not just interested in describing the model posited by Sommerer and Ott; he is interested in the implications of that model. According to the passage, the physicists’ model showed that there could be a system so unstable that valid experimental results could not be duplicated. Answer choice (C) identifies one possible way the system instability could manifest itself: an inability to recreate the starting conditions of an experiment. (A) In the passage, determining the existence of the fractal properties in Sommerer and Ott’s model was never an issue. The significance of fractal properties was that their existence created uncertainty in the behavior of a particle dropped into a riddled basin of attraction. (B) is a 180. The passage expressly states that in a chaotic system, the general destination of a particle can be predicted. (D) and (E) both go far beyond the scope of the passage. A basin of attraction is briefly defined, but the passage never states or suggests that one could usually predict the exact path water will travel if spilled at a point not on the boundary of two ordinary basins of attraction. Likewise, the author never discusses the possibility of determining the path by

Page 10: LSAT_PT 50_Expl_web

which a particle traveled, based upon its starting and ending points. 25. (E) Author’s Attitude Don’t assume the author has a negative attitude. Sommerer and Ott’s work calls into question one of the foundations of scientific research, by suggesting that experimental results could be valid even if it is not possible to reproduce those results. Paragraph 5 is where the author’s attitude is most apparent. The author does not suggest that Sommerer and Ott were mistaken; to the contrary, he suggests that there are potentially many other unstable systems yet to be discovered, and to the extent such systems exist, scientists would be “forced to question one of the basic principles that guides their work.” That’s answer choice (E). (A) starts off with a 180. The author is not skeptical of the possibility that numerous unstable systems exist—he notes that because Sommerer and Ott’s model was the very first one they tried, it is likely that more unstable systems exist. (B) and (D) both head into 180 territory at the end. The author states at the end of the passage that if other unstable systems exist, scientists would be forced to question one of the foundations of scientific research. (C) This one is extreme. The author indicates that it is likely that more unstable systems are waiting to be discovered, but does not express that belief as a certainty. 26. (C) Detail Never try to answer a Detail question without doing your research. Paragraph 3 tells us that in riddled basins of attraction, the path and destination of a particle is uncertain if dropped at the boundary of two basins of attraction. Paragraph 4 distinguishes Sommerer and Ott’s model from a riddled basin of attraction: it’s not just the boundary, but the whole system is such that the path and destination of a dropped particle cannot be predicted. Answer choice (C) captures that nicely. (A) and (B) are distortions—Sommerer and Ott’s system is distinguished from a chaotic system; it is not itself chaotic. (D) and (E) are 180s. The passage specifically states that in a riddled basin of attraction, drops of

water spilled at adjacent points may give the water an entirely different path, velocity, or destination. (Lines 31-36) The passage also specifically states that in the model, the starting point of a particle provides no information at all as to even its general destination. 27. (D) Inference When the author defines a technical term, that definition is likely to figure into a question or two. The author defines “basin of attraction” in paragraph 2, as “the area of land that, whenever water is spilled on it, always directs the spilled water to [a particular body of water].” (Lines 20-22) That’s answer choice (D). None of the other answer choices captures that definition provided in the passage—they all bring in details that do not relate to the meaning of a basin of attraction. 28. (B) Logic (Parallel Reasoning) Sometimes the last question of a set is especially hard. This question is made especially difficult because it comes at the end of a particularly difficult section, it is unusually long for a reading comprehension question, and each answer choice varies only subtly from the others. Stay calm and treat it just as you would any other reading comprehension question, however, and it becomes much more manageable. In paragraph 5, the metaphorical examples of riddled basins of attraction refer to models like the physicists’ model, models of unstable systems for which it is impossible to duplicate experimental conditions. Doing a vertical scan to look for broad differences between the answer choices, we can eliminate (D) and (E) because they do not involve an inability to replicate experimental results. (A) fails because in the repetitions of the experiment, it is possible to exactly replicate the starting conditions of the experiment. (B) captures all of the uncertainties of Sommerer and Ott’s model—results cannot be duplicated because it is impossible to duplicate exactly the starting conditions of the experiment, so it’s our correct answer.

Page 11: LSAT_PT 50_Expl_web

For the record, (C) fails because starting conditions that are approximately the same don’t correspond to Sommerer and Ott’s unstable system. As described in paragraph 1, in the physicists’ model, the tiniest, most inadvertent, imperceptible change in the starting conditions of the experiment would preclude replication of an experimental result. (Lines 4-8)

Page 12: LSAT_PT 50_Expl_web

SECTION 2 LOGICAL REASONING 1. (B) Principle (Formal Logic) Sometimes the LSAT will use an odd question stem for a familiar question type. Ordinarily, we’d expect “most strongly support” in the stem to signal an Inference question. But this is much closer to a Principle question. We have a basic premise in the stimulus (the extract from the lease) and we’ve got to see which of the situations in the answer choices it applies to. And like most Principle questions that ask us to apply the stimulus to the answer choices, that stimulus can be phrased as Formal Logic. If the tenant records preexisting damage on the form, then they won’t pay for it. But if there’s any damage that wasn’t recorded on the form (say, because the tenant caused it), and that damage wasn’t due to circumstances beyond the tenant’s control, then the tenant will pay for it. As long as the tenant fills out the damage form accurately, that sounds pretty reasonable. The stem tells us we’ve got to find a situation where the tenant wouldn’t pay for the damage. That’s (B), where the damage wasn’t on the form but it was caused by a circumstance beyond the tenant’s control. (A) The tenant will definitely pay for this damage, since it wasn’t recorded on the form and it was caused by an event within the tenant’s control. (C) is also a situation where the tenant will pay. The damage wasn’t preexisting, and it would be pretty tough to convince a landlord that the tenant’s children were a circumstance beyond their control (no matter what the tenant might think). (D) The missing light fixture here would be damage caused by the tenant, so it certainly wouldn’t be recorded as preexisting, and the tenant would have to pay. (E) might be tempting, but the lease is designed to force tenants to pay for damage they failed to record. The paint splattering might have been a circumstance beyond the tenant’s control, but recording it on the form wasn’t—the tenant would have to pay here. 2. (B) Weaken the Argument You can weaken an argument by strengthening an opposing argument.

Randy and Marion are having a classic political argument. Randy claims that the mayor’s decision to use her influence in favor of a cable channel was motivated by noble concerns, and Marion counters that the decision was made to reward a political supporter. How could Randy counter Marion’s argument? By shoring up his own argument that the mayor acted to provide diversity of news programming. (B) provides some additional evidence for this argument: the mayor did the same favor for a news service other than Azco, one whose owner supported the mayor’s opponent. (A) The reasons why Azco’s owner supported the mayor are outside the scope. We’re concerned with the reason the mayor did Azeo’s owner a favor. (C) The bias, or lack thereof, in Azco’s coverage is also outside the scope of the argument. (D) Potential sources of political support don’t matter here; Marion’s argument is about past sources of political support. (E) might be true, but it wouldn’t necessarily have anything to do with the mayor’s reasons for supporting Azco. 3. (A) Flaw When you see a survey, study, or experiment in a Flaw question, your first thought should be, “representativeness.” The LSAT loves to build problems around studies that aren’t representative, and this one is no exception. The biologists and trout anglers might have meant well, but there’s plenty wrong with how they conducted their survey. We have no idea how many anglers there actually were, or how many of those anglers caught at least two fish. Most importantly, we don’t know how the anglers chose the fish for the biologists to weigh. If you’ve ever heard the term “fish story,” you’re probably thinking that the fisherman might not have chosen the most representative fish from their catch. Maybe they wanted the biologists to think they were catching big fish, so they turned in the largest of their catch, or maybe they turned in the smallest fish and kept the biggest ones to show off. Either way, they could easily throw off the results of the survey. (A) points out this flaw. (B) The evidence sounds more scientific than anecdotal, since the fish were weighed by a team of biologists. This one isn’t the flaw.

Page 13: LSAT_PT 50_Expl_web

(C), (D) The conclusion of the argument only refers to the weight of trout at the beginning of this season, not over the whole season or as compared to other seasons. (E) Fish other than trout are outside the scope of the argument. 4. (E) Main Point Use the One-Sentence Test to help you answer Main Point questions. If the author had to cut everything out of their argument except for a single sentence, what would they insist on keeping? That single sentence will be the conclusion of their argument, and you can use this test to help you find it. Here, you might have been tempted to pick the first sentence, but it’s really used to support the conclusion in the final sentence, as the Keyword “thus” should help you see. (E) is a perfect paraphrase of that conclusion: “knowing what people value can help predict their actions.” (A) gets it backwards, reversing the author’s conclusion about the correlation cited in the first sentence. (B) People’s claims about what they value are outside the scope of the argument, which cuts right to what people truly value. (C) is far too extreme. The author’s conclusion deals with tendencies, not precise predictions. (D) focuses too much on the details of the example the author uses as evidence, and not on the overall conclusion of the argument. 5. (B) Paradox EXCEPT The correct answer to an EXCEPT question will often be the only answer choice that is outside the scope of the argument. We know from the question stem that four of the answer choices are things that “contribute to an explanation of the difference in health” between the elderly people who stayed in town and the elderly people who recently moved into the community. The correct answer will be the odd man out, the one choice that doesn’t help us explain this paradox. Let’s test each answer choice in turn: (A) tells us why the group that moved into town is in better health—if they’ve got the means to relocate, chances are they’re better off in general. Eliminate.

(B) compares the majorities of two groups, those who’ve moved into the town and those who’ve lived there all their lives. But the paradox in the stimulus deals with two entirely different groups—the elderly who’ve stayed in the town, and the elderly who’ve recently moved there. (B) is outside the scope, and is correct. For the record: (C) if the health care in the community is below average, then those who’ve lived in the community all their lives would have received worse health care than those who moved in recently. This explains the discrepancy—eliminate. (D) The elderly people who recently moved would have changed their environment; according to (D), that would improve their health, explaining the difference. Eliminate. (E), like (A), tells us that people who move are likely to be in better health than those who stay put. Eliminate. 6. (C) Flaw Flaws are closely linked to Assumption questions. A quick glance at the beginning of the answer choices for this question is revealing. “Presumes,” “neglects to consider,” and “ignores the possibility” all suggest that the correct answer will have something to do with the author’s assumption. Finding the assumption will probably help us find the correct answer. This argument is all about architecture and the characteristics of two styles of architecture. The author argues that we could get the best parts of Roman and modern architecture simply by combining some elements of each. If this sounds a little far-fetched, it’s because the author makes a pretty big assumption to get there: that in order to get the effects of two different styles of architecture, it would be sufficient to combine some of the elements of those styles. But combining those elements could just as easily give us a style of architecture that is neither beautiful nor dramatic, as (C) points out. (A) would be an easy trap to fall into—the author assumes that combining elements of the two different styles would be sufficient to create architecture with the qualities of each, but (A) says the author assumes that this combination is necessary.

Page 14: LSAT_PT 50_Expl_web

(B) The author never claims or presumes that the dramatic nature of postmodern architecture is its most salient feature. (D) talks about the interaction between drama and beauty in a single architectural style, but the author talks about these as independent qualities of each style created by certain elements. (E) Other architectural styles are outside the scope. 7. (B) Assumption A new term in the conclusion must be linked to the evidence with an assumption. Finally, someone has gotten around to clinically testing one of the herbal remedies that have become so ubiquitous. And it turns out that the herbal remedy was just as effective as prescription drugs in treating painful joints, so the manufacturer has concluded that chemical agents are unnecessary for treating painful joints. If you were reading carefully, you probably saw that conclusion and thought, “hey, wait a minute. I thought we were talking about herbal remedies and prescription drugs. How did chemical agents sneak in?” That new term, chemical agents, only appears in the manufacturer’s conclusion, so it must be linked to the rest of the evidence by an assumption. We can find this assumption in (B)—the herbal remedy itself contains no chemical agents. If chemical agents really are unnecessary for the treatment of joint pain, then the herbal remedy can’t contain any; otherwise, the whole argument falls apart. (A) It doesn’t matter to the argument whether people will switch to herbal remedies, only whether chemicals are necessary to treat joint pain. (C) The past experience of the people involved in the study is way outside the scope of the argument. (D) We’d certainly be more likely to credit the results of the study if it was performed by the same researchers who analyzed the prescription drugs used for joint pain, but this still wouldn’t mean that herbal remedies and chemical agents aren’t the same thing. (E) All we know about the prescription drug is that it’s effective in treating painful joints. The manufacturer doesn’t care how effective it is, only that his herbal remedy is as effective. (E) is outside the scope of his argument. 8. (A) Paradox (Formal Logic)

Beware of 180’s—they’re one of the most common wrong answer types. The first sentence of this stimulus sets up a simple if/then statement: if an increase in the minimum wage would reduce companies’ profits, those companies often cut their payroll. The paradox appears in the later sentence: even though the fast food industry employs lots of minimum wage workers, a recent increase in the minimum wage didn’t lead to layoffs in the fast food industry. Our job is to find the answer choice that tells us why, but we can do a pretty good job of predicting an answer based on that first if/then. Form the contrapositive: if the companies aren’t laying off workers, then the minimum wage increase probably didn’t reduce their profits. (A) gives us a way this could happen (if decreased turnover cut expenses enough to cover the higher wages), and so it resolves the paradox. (B) might be true, but we’re concerned with the odd situation in the fast-food industry, where jobs weren’t eliminated at all. (C) would actually deepen the paradox; if the fast-food industry acts like all the other industries, we would expect it to reduce its payroll just like they would. (D) The few employees mentioned in (D) wouldn’t necessarily affect the profits of the industry as a whole, especially since the stimulus tells us that most fast-food workers are paid the minimum wage. (E) is another 180. If sales of fast food didn’t increase, that eliminates one possible way that the fast food industry’s profits wouldn’t have been affected by the minimum wage increase. 9. (B) inference (Formal Logic) A valid Inference doesn’t have to take the entire stimulus into account. We can quickly translate the “always” and “never” statements in the stimulus into Formal Logic statements: if a word is the main word in a title, or the first or last word of a title, then it should be capitalized; if a word is an article, a preposition or conjunction with fewer than five letters in the middle of a title, then it shouldn’t be capitalized. We can follow these rules in evaluating the answer choices. Remember the correct answer to an Inference question must be true, while the four wrong answers all could be false.

Page 15: LSAT_PT 50_Expl_web

(A) could be false. The second rule suggests that prepositions or conjunctions could be capitalized in a title if they had five or more letters. Eliminate. (B) must be true. Don’t be fooled by the fact that it doesn’t mention prepositions with fewer than five letters; everything it says still follows logically from the stimulus, even if it could have said more. (B) is correct. For the record: (C) could be false. What if the preposition or conjunction was the main word of a title, or first or last? This might be unlikely, but the stimulus leaves the possibility open. Eliminate. (D) and (E) are both too extreme. The stimulus tells us that certain words in a title should or should not be capitalized. (D) and (E), on the other hand, refer to words in any text. Eliminate them both. 10. (B) Flaw An argument is flawed when its evidence fails to establish its conclusion. As usual, this letter to the editor was written because the author found some fault with the government—in this case, the city council. The letter writer claims that the council’s stated goals in passing an anti-loitering ordinance for the mall weren’t their actual goals. Why? Because the ordinance won’t actually accomplish those goals. While that’s certainly a reason to find fault with the ordinance, it doesn’t mean that the council had to have other goals in mind when it passed the ordinance. Perhaps the council just didn’t realize that the ordinance wouldn’t really do what they thought it would. But the author assumes that for something to be the goal of an action, the action must achieve that goal. (B) points out this flaw. (A) There’s no distinction between primary and secondary goals in the stimulus. In fact, the city council seemed to say that it only had one goal, although that “declared goal” actually encompassed a few different things. (C) The letter writer actually deals directly with the claim’s content, and not the fact that it comes from the city council. (D) starts out well, but then goes off on a tangent about “beneficial effects.” This choice is half right, half wrong. (E) Confusing necessity and sufficient is a common flaw on the LSAT, but it’s not present in this argument.

11. (E) Point at Issue The correct answer to a Point at Issue question must be within the scope of both speakers’ arguments. Don’t be fooled by Cynthia’s opening statement about corporate research. Her real point is that government funding should go to basic scientific research in order to further theoretical understanding. Luis picks right up on this, arguing that government support of research should have an entirely different focus. If Luis got his way, only research that is expected to lead to practical applications in key fields would get government funding. You might be able to predict the exact nature of their disagreement at this point, and it’s fine to take that prediction to the choices, but for now we’ll use the Kaplan Decision Tree to evaluate these choices, asking if each speaker has an opinion on the answer choice and whether their opinions differ. (A) sounds a lot like the middle ground between Cynthia and Luis. Even if we could argue that Cynthia had an opinion on the unforeseen practical applications of theoretical research, something she never mentions, the two speakers would be more likely to agree on this point. Eliminate. (B) Careful—Cynthia says that theoretical research should get government funding, but never says that other projects shouldn’t get government funding. (B) is outside the scope of her argument. Eliminate. (C) The practical benefits of theoretical research are outside the scope of both speakers’ arguments. Eliminate. (D) Cynthia certainly sounds like she’d agree with the statement in this choice. But in Luis’s opinion, government support of research should be limited to where it is likely to yield new technology that benefits society as a whole, not just any new technology. Luis might actually agree with Cynthia on this statement, depending on exactly what kind of new technology would be created. Eliminate. (E) must be correct, simply because we’ve eliminated every other choice. Let’s see why. Cynthia clearly thinks that government research funding should go to basic scientific research regardless of its practical applications, so she’d have a favorable opinion of (E). Luis, however, would disagree with (E), since he thinks new technology must be a likely result of any research the government funds. Both speakers have an opinion

Page 16: LSAT_PT 50_Expl_web

on this choice, and those opinions differ—we’ve found our right answer. 12. (A) Principle A Principle that “most helps to justify” the author’s conclusion will act like a Strengthener in general terms. Short, abstract stimuli can be some of the hardest to decipher on the LSAT, and this one is no exception. But since we know we can approach this question as if we’re trying to strengthen the argument, we can fall back on the strategy of identifying the evidence and conclusion and looking for an assumption. The Keyword “therefore” signals the author’s conclusion: it is impossible to tell whether someone’s action is moral, so we should evaluate consequences instead of morality. The only evidence the author offers for this bold statement is that we can never tell whether someone is acting out of an ulterior motive. This argument needs an assumption to link the mismatched terms: If we can’t tell whether someone has an ulterior motive for their action, we can’t tell whether that action is moral or not. If we were looking for a Strengthener, we’d find an answer choice that provides some support for that assumption. Sure enough, we find that support in (A), which basically restates the assumption in general terms. (B) “Praise and blame” might sound like they’re closely linked to morality, but we can’t let that outside knowledge govern our evaluation of the choices. They’re actually outside the scope of the argument. (C) The argument deals with other people’s actions, not our own. (D) Good or bad people are outside the scope of the argument, which deals only with people’s motives for their actions. (E) distorts the stimulus. The author argues that we should evaluate the consequences of an action instead of its morality, but (E) says that the consequences of an action are essential to evaluating its morality. 13. (D) Strengthen the Argument The correct answer to a Strengthen question will make the conclusion of the argument more likely to follow from the evidence.

Thankfully, we have a strong statement of the author’s opinion to guide us through this question: the fossil-fuel producers’ claim is probably false. The producers’ claim is that reducing carbon dioxide emissions enough to halt global warming would be prohibitively expensive. But the author’s only evidence to refute this claim is an analogy with CFC producers who made a similar claim about the possibility of finding alternatives to CFC’s. The author must assume that the analogy will hold—that there’s enough similarity between the claims of CFC manufacturers and fossil fuel producers to make the analogy stick. To strengthen his argument, we’ll have to find something that bolsters his analogy, and (D) is the only choice that does so. (A) strengthens the argument that environmental regulations were successful in the case of CFC’s, but it doesn’t relate to the author’s claim. (B) could potentially provide some evidence in favor of the author’s conclusion, but it doesn’t withstand scrutiny. The “some countries” it mentions might not be representative, and their reduction in emissions, while not prohibitively expensive, might not be enough to halt global warming. (C) If anything, (C) weakens the argument by suggesting that the analogy between fossil fuel producers and the chemical industry wouldn’t necessarily hold. (E) fails to strengthen the argument because it merely says that the fossil fuel substitutes are possible, not that they wouldn’t be prohibitively expensive. 14. (D) Assumption (Formal Logic) The author’s assumption will link the evidence to the conclusion. This author voices a perennial complaint: contemporary legislatures don’t enact laws that benefit their constituents. The evidence takes several steps in doing so, but it gives us a good reason why—if legislators want to enact laws that benefit their constituents, they must consider the consequences of enacting those laws. (Note that we can use the contrapositive to link this piece of evidence more directly to the conclusion: If legislators don’t consider the consequences of enacting particular laws, then they will not enact laws that benefit their constituents.) Contemporary legislators, on the other hand, are so concerned with politics that they only frame legislation in polemical terms, which arouses either repugnance or enthusiasm in their colleagues.

Page 17: LSAT_PT 50_Expl_web

Like many tougher Assumption questions, this one uses difficult language to hide the fact that all we have to do is put together the missing pieces of the argument. Here, the gap is between what the legislators do and what the author thinks they should do, or between presenting legislation polemically and looking at the consequences of enacting the legislation. To contrast these two actions, the author must assume that legislators don’t do both, as (D) says. (A) certainly sounds like an inference we could draw from the author’s argument, but that’s not what we’re looking for. (B) and (C) Successful legislatures are outside the scope of the author’s argument, as is constituents’ adherence to the laws. (E) is close, but goes a bit further than the author’s actual assumption. The author assumes that legislators don’t consider the consequences of laws that arouse either repugnance or enthusiasm, not that the strong emotions actually prevent them from considering the consequences of the legislation. 15. (E) Role of a Statement When a statement is neither evidence nor the conclusion, you must determine how it relates to the author’s argument. Although it’s a bit difficult to determine exactly how the statement in question plays into the argument, we’re aided by the fact that the argument itself is rather simple. Anderson thinks that travel writing has gotten worse over the last few decades, and the author disagrees. The author supports his conclusion by arguing that travel writing is still acclaimed by a much more discerning public than it reached in the past. The fact that the public is better traveled than it used to be is the statement in question, and it supports the author’s conclusion by giving us a condition under which today’s travel writers must operate, as (E) says. (A) The author never claims that there is a causal relationship between travel writing and the travel experience of the public. (B) The intended audience of contemporary travel writing is outside the scope of the argument. (C) is a 180—the author states that today’s readers enjoy contemporary travel writing. (D) The statement in question does relate to the author’s contention that travel writing flourishes

more now than it does in the past, but it is stated as an obstacle, not a cause. 16. (C) Assumption Connect mismatched terms in the argument to find the author’s assumption. The author’s conclusion sets up a proportional relationship for multiparty democracies: the fewer the parties, the more productive the legislatures. The author supports this conclusion by delving further into proportions. Fewer parties means more issues for each party, and more issues means prioritization, which means compromise. But that’s where the author’s evidence ends, and it’s still short of explaining how exactly fewer parties means more productive legislatures—unless we use an assumption to link the mismatched terms. The evidence gets us all the way from fewer parties to a greater tendency to compromise. We can get the rest of the way to the conclusion with one more step: a greater tendency to compromise leads to more productive legislatures, (C). (A) The evidence actually suggests that more political parties would each take stands on fewer issues, which might actually lead to less disagreement within parties. (B) The importance of compromise isn’t part of the author’s argument; the tendency to compromise is. (D) Nondemocracies are outside the scope of the argument, which deals only with multiparty democracies. (E) The author’s argument is all about compromise within legislatures, not unanimity. 17. (A) Parallel Flaw Finding the flaw in a Parallel Flaw stimulus can help you quickly find the correct answer. The evidence in this stimulus sets up two correlations: when air is warmer, it is more humid, and when it is more humid, there is more rain. But the author draws a conclusion from this evidence that is way off base (as we’d expect, since the question stem told us that the argument was flawed): increased rainfall must have been the result of warmer air. This conclusion gets the relationship implied by the correlation backwards, and in so doing, neglects a whole host of alternative possibilities that could lead to increased rainfall, so the correct answer must neglect alternative possibilities. (A) is a perfect parallel, setting up two

Page 18: LSAT_PT 50_Expl_web

correlations (fresh food is more nutritious, nutritious food leads to better health), reversing the implied causal relationship (better health must mean fresher food) and ignoring all the other things that could have led to the result in the conclusion. (B) is a mere tautology. More importantly, it doesn’t set up the double correlation found in the stimulus. (C) The two correlated factors in (C) actually seem like a single correlation phrased two different ways—carrying lots of bags and packages makes travel on mass transit difficult and unpleasant. The flaw in this argument is the unwarranted assumption that this difficulty will lead most travelers with packages to ride in autos instead. (D) mistakenly applies statistics about a group to every member of that group, which is again a different flaw than that found in the stimulus. (E) There is only a single step of correlation in the evidence of (E), so we can eliminate it without continuing on to the conclusion. 18. (C) Inference Beware of extreme language in Inference answer choices. The experiment in this question teaches us about the brain functioning of toddlers—apparently, most toddlers are unable to adapt to new rules because their prefrontal cortex is insufficiently developed. This part of the brain continues to develop until adolescence and is essential for certain functions, including adapting to new rules. The key to answering this Inference question (and many others) is to notice the qualification—the toddlers almost invariably failed to pick up the new task. That “almost” means that the failure to adapt to new rules wasn’t universal, so we can eliminate any answer choices that suggest it was. (A) The stimulus suggests that almost all toddlers cannot sort objects by color after they first sorted the objects by shape, but not that their prefrontal cortexes are thus less developed than those of other toddlers. Eliminate. (B) Is too extreme—the “almost” in the stimulus suggests that some toddlers were able to solve the problem that required adapting to a new rule. Eliminate. (C) ‘Disobedient behavior’ may sound like it’s outside the scope at first glance, but it’s a pretty good characterization of failing to follow instructions.

The stimulus suggests that toddlers’ failing to follow instructions when those instructions include a new rule is the result of an underdeveloped brain, not disobedience. (C) must be true, and is correct. For the record: (D) The relative importance of upbringing and the maturing of the prefrontal cortex are outside the scope. Eliminate. (E) is also too extreme. While the development of the prefrontal cortex is essential to adapting to new rules, the stimulus never suggests that the development is proportional to the level of skill in adaptation. Eliminate. 19. (A) Role of a Statement Keywords can help you determine the Role of a Statement. The dietitian’s argument is designed to refute a very narrow misconception within a larger concept. She starts us off with the two premises of that misconception: nutrients are more effective when provided by natural foods instead of supplements; and dietary fat is generally unhealthy. But like most general rules, these have exceptions. The author’s example, eating carrots, shows us a specific exception to the ‘fat is bad’ rule—our bodies need some fat to turn beta carotene into vitamin A. This example leads us to the dietitian’s conclusion, which isn’t explicitly stated in the argument: there is an exception to the general rule about dietary fat, especially when getting your nutrients from natural foods. It sounds like the author wants us to adopt a specific dietary practice (eating some fat with our carrots), and that the statement in question encourages us to do so, as (A) suggests. (B) “Those who recommend” the dietary practices are outside the scope, since the dietitian doesn’t refer to any such recommenders. (C) distorts the author’s position. She doesn’t undermine the rule completely; she just says that it doesn’t apply in every situation. (D) The author doesn’t attack the claim, she uses it for support. (E) also distorts the stimulus. The dietitian does recommend a dietary habit, but the statement in question is a rule to which the habit creates an exception, not a bad reason for adopting the habit. 20. (C) Main Point

Page 19: LSAT_PT 50_Expl_web

Don’t get bogged down in the details in Main Point questions. The engineer does a good job of hiding his opinion until later in the stimulus. First he talks about the suggestion of some other people that we should address the problem of global warming by pumping carbon dioxide into the deep ocean, then he talks about environmentalists’ objections to this suggestion. We don’t arrive at the author’s opinion until the third sentence of this long stimulus, where he tells us that the environmentalists’ concerns are unfounded. (C) restates this conclusion. (A) endorses the suggestion put forward in the first sentence, but the author never explicitly does so. He only suggests that one objection to this suggestion is unfounded. (B) recites the details of the second sentence, the environmentalists’ objections. (D) is another detail, this time from the author’s evidence in the final sentence. (E), like (A), goes a bit too far. The author never says that the suggestion in the first sentence is a good one or that it should be considered, only that one particular objection to the suggestion is unfounded. 21. (E) Paradox Beware of answer choices that contradict the stimulus. The language in this question stem, “resolve the apparent discrepancy,” is a classic signal for a Paradox question. The discrepancy in the stimulus is between certain people at a seafood restaurant on a certain date: some of them became ill from a type of bacteria found in contaminated seafood, but most of the customers of the restaurant on that date hadn’t come into contact with the bacteria. Yet health officials were still confident that contaminated seafood from the restaurant had caused the illness, and therein lies the problem. Why would health officials be sure that contaminated seafood from the restaurant had caused the illness when most of the restaurant’s customers hadn’t come into contact with the bacteria that caused the illness? Well, as (E) suggests, perhaps only a particular type of seafood was contaminated, affecting only those customers who ate it. (A) If everyone in the restaurant had been exposed to the bacteria, then (A) would explain why most of

them weren’t sick, but that’s not the situation we’re dealing with. (B) doesn’t help to explain why only certain people became sick—if the illness is caused by bacteria in seafood, coming into contact with someone who later became ill shouldn’t have any effect. (C) contradicts the stimulus, suggesting that the illness was caused by MSG instead of bacteria. (D) might make us more likely to believe that the restaurant would make people sick, but it doesn’t give us a specific reason why the health officials were so sure that contaminated seafood from the restaurant caused the illness. 22. (D) Assumption (Formal Logic) The author’s central assumption is an unstated but necessary condition of their argument. The words “only if” in the first sentence of the economist's argument are a gift whenever they appear in an Assumption question. The same is true of any Formal Logic phrase, because Formal Logic helps us find what’s necessary to the author’s argument, and that will always help us find their assumption. The economist tells us that real wages in the country will rise only if productivity increases notably. Replacing “only if” with “then,” we get our simple Formal Logic statement: if real wages in the country go up, then productivity must have done so as well. We can quickly form a contrapositive: if productivity doesn’t increase notably, then real wages in this country aren’t going to rise. But the economist then concludes that wages aren’t going to increase, because businesses in the country aren’t investing in new technology and aren’t going to do so in the near future. Well, if that’s the case, then investments in new technology must be linked to productivity, since we already know that increased productivity is necessary for increased wages. In fact, we can tell that the economist assumes that investment in new technology is necessary to bring about an increase in productivity, and (D) restates this proposition. (A) The economist doesn’t make any arguments about the last several years. Her argument only deals with what is likely to happen in the next few years. (B) actually contradicts the economist’s argument. If an increase in productivity is necessary for an increase in wages, then a significant number of

Page 20: LSAT_PT 50_Expl_web

workers acquiring the skills necessary to use new technology won’t be sufficient to increase wages. (C) The economist might believe wages will increase sooner or later, but it’s not central to her argument. (E) Even if the new technology in which businesses are currently investing is contributing to an increase in productivity, the economist still implies that there isn’t enough investment to notably increase productivity. A less-than-notable increase presumably wouldn’t be enough to affect productivity. 23. (E) Principle (Parallel Reasoning) A Principle question that requires you to find a situation that uses the same principle can be attacked as if it’s a Parallel Reasoning question. A Parallel Flaw question requires you to find the argument that has not only the same type of evidence and the same type of conclusion, but also the same type of flaw. Parallel Principle questions are very similar in that they require you to find an answer choice that uses the same principle as the stimulus. Just like it’s easiest to attack Parallel Flaw questions by examining the flaw, it’s best to attack Parallel Principle questions by first finding the principle. Here, the author is concerned with conflicts of interest in scientific journals. The scientists who write these journals praise certain companies, but it turns out that the same scientists have substantial investments in those companies. In other words, they have something to gain if the companies do well, so we can’t necessarily trust that their praise is warranted. The author suggests a way of dealing with these conflicts of interest. His conclusion is that the scientists should disclose their investments. Note that this doesn’t go as far as we might expect; the author doesn’t suggest that the scientists should, for example, sell their investments in companies they discuss in their journals. He only wants the scientists to let everybody know that they have a potential conflict of interest. We’ll take that into account in formulating our principle: Anyone whose integrity could be called into question by a conflict of interest has a duty to disclose that conflict of interest. We can use this principle to evaluate the choices. (A) goes further than the author does, suggesting that managers should avoid conflicts of interest altogether. Eliminate. (B) The author’s principle doesn’t deal with the claims the scientists make, it only deals with the

scientist's actions surrounding those claims. Eliminate. (C) doesn’t suggest anything about a conflict of interest; instead, it argues that we should ban insider trading. Eliminate. (D), like (A), argues in favor of banning certain conflicts of interest. This is more extreme than the author’s position. Eliminate. (E) Finally, we find another situation where a group giving advice should be required to disclose any conflicts of interest. (E) follows the same principle as the stimulus, and is thus correct. 24. (A) Weaken the Argument EXCEPT Any argument that makes a prediction assumes that present circumstances will hold true in the future; we can weaken that prediction by showing that circumstances will change. The columnist sets up a simple causal relationship between acidic pollutants and acid rain. The amount of acidic pollutants going into the air has decreased, so, according to the columnist, we can expect that the negative effects of acid rain will also decrease in the coming years. Well, that may be the case, but there must be something wrong with this argument, since we’re all set to weaken it. The problem is actually common to every argument that makes a prediction. We can never be absolutely certain what will happen in the future, so we have to base predictions on the circumstances of the past and present. Any prediction thus assumes that those circumstances will continue to hold true. Anything that gives us a reason to believe that circumstances will change in the future will weaken the argument. We know from the EXCEPT in the question stem that four of the answer choices will give us some reason to think that circumstances will change and make negative environmental effects more likely, but the correct answer will not. (A) doesn’t give us any reason to think that the negative effects of acid rain will get worse in the future. In fact, it suggests that the negative effects won’t be so bad in some ecosystems, so (A) actually Strengthens the argument. This is our correct answer. For the record: (B) If there are fewer buffers in the air neutralizing acid, then the acid that’s present in the air would be more destructive. (B) gives us a reason to think that past circumstances won’t hold, so it weakens the argument. Eliminate.

Page 21: LSAT_PT 50_Expl_web

(C) If the decrease in acidic pollutants is set to turn into an increase, then the negative effects of acid rain will quickly stop getting better. (C) weakens the argument and can be eliminated. (D) gives another reason why the decrease in acidic pollutants may not lead to less environmental damage from acid rain. Eliminate. (E) tells us something similar to (B)—some factor in the environment that mitigated the effects of acid rain is going away, so the damage caused by acid rain will likely be worse in the future. Eliminate this choice, too. 25. (D) Principle Find the principle illustrated by an argument by searching for a law-like rule that sums up the author’s point. Morality is a tricky thing, and the columnist gives us one reason why: our moral judgment of an action is often colored by external circumstances. This statement is the columnist’s conclusion, and it refutes the idea that a person’s intentions are the only relevant factors in determining the morality of their actions. The example of the absentminded cook is used as an illustration to further the columnist’s point. The cook wouldn’t be morally blameworthy for putting an odd ingredient in his stew, unless that action caused some great harm. But the blameworthiness of his action is completely independent of what he did—it depends entirely on circumstances beyond his control, in this example the allergies of someone eating his cooking. We’ll have to factor this example into our expression of the principle: moral blameworthiness can sometimes be the result of circumstances beyond our control. (D) accurately expresses this principle. (A) is the opposite of the author’s conclusion, stating that moral judgments can be made without considering all the circumstances. Eliminate this 180. (B) Even though the columnist seems to hint that the principle behind his argument is a bit odd, he never suggests that it’s unfair. (C) is another 180. This is the view expressed in the first sentence of the stimulus, and it’s the view that the author tries to refute. (E) is a bit too extreme. We don’t know the frequency or rarity of the types of circumstances that would cause us to change our moral judgment.

Page 22: LSAT_PT 50_Expl_web

SECTION 3 LOGIC GAMES Game 1: Traveling by Air Situation: A traveler is choosing airlines at each of six stops. Entities: The airlines: L, M, N and O Action: Sequencing Limitations: One airline per stop. Notice that the LSAT never says that the traveler must use every airline! The Initial Setup: L M N O ___ ___ ___ ___ ___ ___ 1 2 3 4 5 6 The Rules: Rule 1 tells us that our traveler may not choose the same airline at any two consecutive stops. We could write this a couple of different ways: No Consecutive Pairs Never LL, MM, NN, or OO; we’ll stick with this one because it’s more precise. Rule 2 says that the traveler chooses the same airline at stops 1 and 6. Again, we could use a couple of different shorthands for this rule. We could draw an arrow to each slot, or we could just write “1=6” off to the side. Let’s do the latter for now: 1 = 6 Rule 3 gives us the same shorthand options as Rule 2, saying that the traveler chooses the same airline at stops 2 and 4. Again, we’ll pick the simpler shorthand: 2 = 4 Rule 4 says that if L or M is chosen at a stop, then N cannot be chosen at the following stop: Never LN Never MN Rule 5 says that the traveler chooses N or O at Stop 5. We can draw that right into our sketch. In fact, this rule leads us to our first Deduction. We can draw two sketches here, one for each entity in the fifth stop:

Option 1: ___ ___ ___ ___ N ___ 1 2 3 4 5 6 Option 2: ___ ___ ___ ___ O ___ 1 2 3 4 5 6 Deductions: Let’s continue with these. As is usually the case, the rules allow us to make several deductions for at least one option. In Option 1, we can use a Duplication to get further. Neither L nor M can come before N (Rule 4), and N can’t come before itself (Rule 1). So O is the only airline that can be chosen for slot 4, and that means it also must be in slot 2. Keep filling in Deductions for Option 1: Rule 1 tells us that we can’t put O in slot 1 or slot 3, and we can’t put N in slot 6. Because we must choose the same airline for slots 1 and 6, and neither N nor O can be chosen for those slots, they must be either L or M. We can fill that into our sketch as well. We could be content with “no O” in slot 3. However, we’ve got everything else filled in, so let’s go ahead and turn the negative rule into a positive. Slot 3 can be filled with either L, M or N, giving us this final sketch for Option 1: L/M O L/M/N O N L/M 1 2 3 4 5 6 Now let’s work on Option 2. Unfortunately, we can’t do much else here because O has no restrictions in which airline can either precede or follow it—other than Rule 1. Rule 1 tells us that O cannot be placed in the 1/6 or 2/4 slots, but that’s about it. We still have several different options with respect to the remaining slots, and so we should leave those slots open until we get a question that forces us to fill them in. The Final Visualization: Airlines: LMNO Never LL, MM, NN, or OO 1=6 2=4 Never LN or MN Option 1: L/M O L/M/N O N L/M 1 2 3 4 5 6

Page 23: LSAT_PT 50_Expl_web

Option 2: L/M/N L/M/N ___ L/M/N _O_ L/M/N 1 2 3 4 5 6 The Questions: 1. (D) Acceptability Use the rules to eliminate the unacceptable hypotheticals quickly. Rule 1 says that the traveler cannot choose the same airline at any two consecutive stops, which eliminates (A). Rule 2 requires the same airline to be chosen for stops 1 and 6, which rules out (E). Rule 3 requires the same airline to be chosen for stops 2 and 4; (B) violates this rule, so we can eliminate it. Rule 4 prohibits us from choosing N after any stop where L or M was chosen, which takes out (C). (D) is left over, and is correct. 2. (B) “If” clause / “could be true” Where you have drawn Limited Options, expect an “if” question to put you in one option or the other. Our new “if” puts us squarely into Option 1—in fact, it’s how we made our Options in the first place. We can just check the answer choices against Option 1 in our Master Sketch. Because the traveler must choose O for stop 2, we can quickly eliminate answer choices (A), (C) and (D). (E) puts N in stop 1, which is not permitted in this option—it would also mean that N was in stop 6, which violates Rule 1. (B) is the only choice remaining, so it must be correct. Looking back at our sketch, it works perfectly. 3. (C) “If” clause / “must be true” Even with Limited Options, you must be willing to quickly redraw your sketch to work out the Deductions from a new “if” rule. Here we are given two new “if” rules: The traveler chooses only airlines M, N and O for the trip, and she chooses O for the 5th stop. The first new “if” might have thrown you for a loop if you hadn’t been strict about the Limitations, but it actually makes our work easier: we don’t have to deal with L at all. The second new “if” puts us squarely into Option 2, so we can start by quickly recopying that sketch. Because we’ve eliminated L from this question, we can change the L/M/N in the 1/6 stops and the 2/4 stops to M/N—one or the other of those airlines must be chosen for all four of those stops.

Rule 4 prevents N from immediately following M. The only place this would be an issue is in stops 1 and 2—we can’t have M in 1 and N in 2, so we must have the reverse. Stops 1 and 6 must be filled by N, and stops 2 and 4 must be filled by M,. Because N cannot follow M, and we cannot choose M for two consecutive stops, O is the only viable choice for stop 3. Our sketch, with everything filled in, looks like this: N M O M O N 1 2 3 4 5 6 Therefore, (C) is the correct choice—stops 1, 2, and 3 are N, M, and O, respectively. (A) doesn’t have N after M, but if O is in stop 2, it must also be in stop 4—this would violate Rule 1. (B) violates Rule 4—N cannot be chosen immediately after M. (D) has the same problem as (A). (E) has a similar problem. If O is in stop 1, it must also be in stop 6, but this would violate Rule 1. 4. (E) “CANNOT be true” A Limited Options set-up can often provide a quick answer for a not-if question. When the correct answer is something that “cannot be true,” the four wrong answers are all possible. Usually, we can use sketches from previous questions to test the choices. If we’ve seen one of the answer choices in our previous work, we know it’s possible and we can eliminate it. Here, our options will work just as well as previous sketches. (A) Option 1 puts O in stop 2, so if M is there, we’re in Option 2. Nothing in this option suggests that we couldn’t put L in stops 1 and 6 and M in stops 2 and 4. Eliminate this choice and try the next one. (B) and (D) If O is in stop 2, we’re in Option 1 and only L or M could be in stop 1. (B) chooses L, and (D) chooses M; both of these are possibilities and can be eliminated. (C) Back to Option 2 for this one: there’s no reason it couldn’t work, although it would mean we couldn’t ever select N—if you assumed that every airline had to be used, you might have picked (C). Eliminate it instead. (E) If we try putting O in stop 1, we must also put O in stop 6, which means that O could not be in stop 5 (Rule 1). But if N is in stop 2, N is also in stop 4, and

Page 24: LSAT_PT 50_Expl_web

N could not be in stop 5. This would violate Rule 5, which requires our intrepid traveler to choose N or O for stop 5. Thus (E) cannot be true, and is the correct answer. 5. (B) “If” clause / “could be true” If you missed the Limited Options in a game, methodically applying the rules to a “new-if” question will get you to the right answer. The new-if immediately puts us into Option 1, which tells us that slot 5 must be held by N and slot 6 can be filled with either L or M, quickly taking us to correct answer choice (B). Had we not drawn out the limited options in our initial set-up, we would re-copy our sketch, apply the new “if,” and redo step 4 of the Kaplan Method, like so: If O is in slot 2, then O must also be in slot 4 (Rule 3). This eliminates O from slot 5, which rules out answer choices (D) and (E). Rule 5 tells us that slot 5 must be filled with either N or O. If O is out, N is in, and that rules out answer choice (A). Rule 2 tells us that slots 1 and 6 are filled by the same airline, which means that O cannot be placed into slot 6—it would also be in slot 1, which would violate Rule 1. That rules out answer choice (C), and leaves us with correct answer choice (B). The scratch work ends up just like our Option 1: L/M O L/M/N O N L/M 1 2 3 4 5 6 Game 2: PTA Situation: A school committee Entities: Parents: F, G, H Students: k, l, m Teachers: U, W, X, Z Action: Selection Limitations: 5 of 10 candidates are selected, which means that 5 are rejected. Keep an eye on the numbers selected and rejected from each subgroup. The Initial Setup: The inclusion of subgroups of candidates makes this game slightly more complicated than a straightforward selection game. We can be sure that the subgroups will play a role in the game, and with half of the entities chosen and half rejected, keeping track of who gets rejected can be very important. We can adapt our usual strategy of making a roster by separating the subgroups.

Make sure you write each subgroup of entities differently, like so: Parents: students: Teachers: F, G, H k, l, m U, W, X, Z The Rules: Rule 1 says that the committee must include exactly one student. We can indicate that in our sketch: Parents: students: Teachers: F, G, H k, l, m U, W, X, Z Exactly 1 Rule 2 tells us that F and H cannot both be selected. This gives us an important Numbers deduction: we can never choose all three parents. We can make a note: Never FH Rule 3 tells us that M and Z cannot both be selected. Thus, we note: Never mZ Rule 4 prohibits U and W from serving on the committee together. Here’s another Numbers deduction: we never have all 4 teachers: Never UW Rule 5 brings in a formal logic rule: F cannot be selected unless Z is also selected. Translate this rule and form the contrapositive: If F Z If no Z no F Rule 6 gives us another “unless” rule. “W cannot be chosen unless H is chosen” gets translated as: If W H Contrapositive: If no H no W Deductions: We have no Blocs of entities that must be chosen together. The other usual way we can get Limited Options is through consideration of the Numbers. The first two rules plus Rule 4 limit the numerical possibilities for our committee to two options: Either we’ll select two parents, one student, and two teachers, or we’ll select one parent, one student, and three teachers.

Page 25: LSAT_PT 50_Expl_web

Parents: students: Teachers: F, G, H k, l, m U, W, X, Z 2 1 2 1 1 3 Rule 4 tells us that if we select three teachers, they must include X, Z, and one of U or W. Rule 2 gives us a similar deduction: if we select two parents, G must be selected along with either F or H. However, there are many possibilities for the other committee members in either of our two options, so we won’t take the time to draw them out. We have no Established Entities, but we do have some Duplications in our rules that are worth considering: Rule 5 tells us that F requires Z, but Rule 3 tells us that Z and M cannot both serve on the committee. Thus: If F no m And the contrapositive: If m no F or we could simply note: Never Fm Likewise, Rule 6 tells us that W requires H, but Rule 2 says that F and H cannot both serve on the committee. Thus: If W no F If F no W Or Never WF Final Visualization: Parents: students: Teachers: F, G, H k, l, m U, W, X, Z 2 (G, F/H) 1 2 1 1 3 (X, Z, U/W) Never FH Never mZ Never UW Never Fm Never WF F Z W H No Z no F No H no W The Questions: 6. (E) Acceptability

Don’t worry if some of the rules don’t eliminate any choices in the Acceptability question; just keep moving confidently through them. Rule 1 eliminates (A). Rules 2 and 3 are not violated in any of the answer choices, so we’ll leave them and move on. Rule 4, however, knocks out (D), Rule 5 takes care of (B) and Rule 6 gets rid of (C), leaving (E) as the clear winner. 7. (D) “If” clause / “Could be true” Use the new “if” rule to make new deductions; they’ll tell you the answer to the question. If W is selected, Rule 4 tells us that U is out, and Rule 6 tells us that H is in. Once that’s in our sketch, Rule 2 tells us that F is out. That eliminates (A) and (E). If Z is chosen, the rules say that m is out, and so is (C). K and L are both students. Rule 1 tells us that we only get to choose one student for the committee, so (B) is wrong. That leaves (D) as the correct answer. 8. (B) “CANNOT be true” Before you try to eliminate answer choices by drawing out hypothetical groupings, check to see if you can spot the bad pair by applying the rules and your deductions. With no obvious reason to avoid grouping F and G as (A) suggests, we move on to (B). We already deduced that F and M never get selected together, and so (B) has the pair that cannot both be chosen. For the record: (C) and (D): Both of these pairs include a parent and a student, neither of whom trigger any of the rules dealing with the teachers. We could select U, X, and Z to fill out these selections with no trouble. (E) If m and U are selected, Z and W are rejected, so X must be selected and F must be rejected. We’ve already got our students and teachers, and we can fill out the selection with the remaining two parents, G and H. 9. (A) “If” clause / “Could be true EXCEPT” If the wrong answers all could be true, you can use your previous work to eliminate them. Look back at our selection for Question 7: we could select G or l with W, so we can quickly eliminate (B) and (C). The same goes for (E): we’ve already seen

Page 26: LSAT_PT 50_Expl_web

Z and W selected together. We can quickly work up a sketch to eliminate the last wrong choice, and once again, a deduction helps us pick up a quick point. We already noticed that if W is chosen, H must be chosen, and if H is chosen, F cannot be chosen and (A) is correct. Because m is still available to be selected, we can cross off (D) just to be sure. 10. (E) “If” clause / “Must be true” If you are able to make a Numbers deduction, expect a question about the arithmetic. If the committee is to include one parent, the other members must include one student and three teachers. Our initial setup tells us that we must select X and Z in this case, so (E) is the correct answer. For the record: (A), (B), and (C) are all students: as long as we have one of them, our selection is fine, and picking a single parent won’t tell us which student is chosen. (D) Even with three of the four teachers selected, we could choose either U or W. 11. (B) “If clause” / “Must be true” Rejecting an entity is just as helpful as selecting it. If m is selected, Z is out. If Z is out, then F cannot be selected. Not only that, but we’ve got to select two teachers because if we selected three teachers we’d have to select Z. We can’t select both U and W, but we’ll have to select one of them along with X. Selecting two teachers means that we’ll also select two parents: with F knocked out, that means we’ll select G and H. M is a student, so K and L are also rejected. We can now select or reject almost every entity: Parents: students: Teachers: F, G, H k, l, m U / W, X, Z (B) is correct: G and H must be selected. For the record: (A) F was one of the first entities we rejected. (C) and (D) With m selected, k and l are both eliminated.

(E) U and X could be selected, but so could W and X. We have no way of telling which of these pairs is selected, so (E) doesn’t have to be true. Game 3: Grads and Cars Situation: Friends graduating and buying cars. Entities: Friends R, S and T. Action: To sequence the graduation and first car purchase by each friend: a Double Sequencing game. Limitations: Each event takes place over a period from 1991-1995. Each friend graduates (once) and buys his/her first car (once) during one of those years. Note that there are no limitations regarding the number of friends who could graduate or purchase a car in any particular year. Note also that there are five years, but only three entities, so there will be some blanks in each sequence. The Initial Setup: Each friend experiences two events: graduation and the purchase of a new car. We can anticipate that sometimes we will be given information about the year of a friend’s graduation, and other times, we’ll get information about the year in which the friend purchased his or her first car. We’ll need to include space for each event in our sketch, and write out our set of entities once for each event: Grad: ___ ___ ___ ___ ___ R, S, T Car: ___ ___ ___ ___ ___ R, S, T 91 92 93 94 95 The Rules: Rule 1 says that Ramon graduated in some year before the year in which Taylor graduated. Therefore, Grad: R . . . T Rule 2 tells us that Taylor graduated some year before the year in which he bought his first car. We can combine this rule with Rule 1: Grad: R . . . T . . . Car: T Rule 3 gives us information about Sue -- she bought her first car some year before the year in which she graduated:

Page 27: LSAT_PT 50_Expl_web

Grad: S Car: S . . . Rule 4 brings our scratchwork together, and narrows the possibilities by quite a bit – R and S graduate in the same year as each other: Grad: RS . . . T . . . Car: S . . . T Rule 5 narrows the possibilities much further. Now we know that at least one of the friends graduated in 1993. There are only two possibilities: either Ramon and Sue graduated in 1993, or Taylor graduated in 1993. The year in which Ramon bought his car is a floater. Deductions: We made several Deductions based on Duplications as we went through the rules. That’s fine, as long as we keep looking for further Deductions during Step 4. We ended up with one gigantic Bloc of Entities, and Rule 5 set part of this Bloc into Limited Options, so we’ll draw them out. Option 1: If Ramon and Sue graduated in 1993, then Taylor had to graduate in 1994 and purchase his first car in 1995. Sue must have purchased her first car in either 1991 or 1992. Option 2: If Taylor graduated in 1993, then Ramon and Sue had to graduate in 1992, and Sue must have bought her first car in 1991. Taylor bought his first car in either 1994 or 1995. Final Visualization: Option 1: Grad: ___ ___ RS T ___ Car: ___ ___ ___ ___ T R 91 92 93 94 95 S Option 2: Grad: ___ RS T ___ ___ Car: S ___ ___ ___ ___ R 91 92 93 94 95 T

The Questions: 12. (B) Partial Acceptability Limited Options can make an Acceptability question particularly manageable. Because we were able to fill in so much information on our Limited Options, we can use those options to answer this partial acceptability question. In Option 1, Ramon and Sue graduated in 1993 and Taylor graduated in 1994. That’s answer choice (B). Going through the rules, we can eliminate choices just like in other Acceptability questions. Rule 1 eliminates (D), because Ramon must graduate before Taylor. Rule 2 eliminates (E): If Taylor bought his car some year after he graduated, he couldn’t graduate in 1995 and still have time to buy his car. Rule 3 knocks out (A) for a similar reason—Sue couldn’t have bought her car before she graduated if she graduated in 1991. Finally, Rule 4 eliminates (C), because Ramon and Sue graduate in different years in this choice. 13. (B) “Could be true” It’s often easier to eliminate wrong answer choices than it is to identify the correct answer immediately. If the right answer is something that could have happened in 1995, then the four wrong answers are all events that could not have happened in 1995. We know from our Limited Options that none of the friends could have graduated in 1995, so we can quickly eliminate answer choices (A), (C), and (E). Sue bought her car before she graduated, so she could not have bought her first car in 1995, eliminating (D). We might even have been attracted to correct answer choice (B) right off the bat, as Ramon’s first car purchase is the only floater in the game. Often, anything said about a floater can be true. If you were running short of time, you might have just jumped straight to (B), because it’s very likely that a statement about a floater could be true. 14. (C) “Must be false” Drawing out Limited Options saves time on “Not-If” questions. If the right answer choice must be false, then the four wrong answer choices all could be true. We can evaluate them in order, checking the possibilities against our Options.

Page 28: LSAT_PT 50_Expl_web

(A) and (B): Both Sue and Ramon could have bought their first cars in 1991 in either Option, or in 1992 in Option 1. Eliminate these choices. (C) The only friend who could have purchased a car in 1993 is Ramon. Thus, answer choice (C) must be false and is the correct answer. For the record: (D) and (E): Both Taylor and Ramon could have bought their first cars in 1994 or 1995, as seen in Option 2. 15. (A) “Must be true” Don’t waste time checking the wrong answer choices once you’ve found a choice you’re sure is correct. If the right answer must be true, the wrong answers all could be false. Once again, our Limited Options make short work of these choices. We’ll evaluate the answer choices in turn: (A) Our Limited Options show that the friends must have graduated either in 1993 and 1994 (Option 1) or 1992 and 1993 (Option 2). Thus, none of them could have graduated in 1991, and answer choice (A) must be true. We could compare each of the other answer choices to our two options to see if they could be false, but because the LSAT writers only give us one right answer choice and four rotten ones, why bother? On Test Day, once you’re sure that a choice is correct, you should move on. For the record, here are the reasons why the last four answer choices are incorrect: (B) Ramon and Sue could have graduated in 1992 in Option 1. (C) and (E) Ramon could have bought his car in any of the five years. (D) Taylor could have graduated in 1994 in Option 2. 16. (E) “If” clause / “could be true EXCEPT” Take time to characterize the choices when you see an “EXCEPT” in the question stem. Our new “if” has Taylor graduating in the same year that Ramon bought his first car. Because Taylor must have graduated either in 1994 (Option 1) or 1993 (Option 2), those are now the only possible years in which Ramon could have purchased his first car. The question stem tells us that each of the

wrong answer choices could be true, and the one right answer choice must be false. (A) Sue could have bought her first car in 1991 as long as we’re in Option 1. Eliminate this choice. (B) Ramon could have graduated in 1992 in Option 2. Eliminate this one too. (C) Taylor could have graduated in 1993 in Option 2. Eliminate. (D) Taylor could have bought his first car in 1994 in Option 2. Eliminate. We have high hopes for answer choice (E), and indeed, (E) must be false: when we incorporate our new if, Ramon could not possibly have purchased his first car in 1995. Once we’d made a Deduction about Ramon, it would have been a good idea to jump to (B) and (E), because those choices mention Ramon. Focusing on our Deductions will always save time. 17. (E) “If” clause / “Must be true” When the correct answer must be true, any choice involving a floater is usually going to be wrong. Our new “if” puts us into Option 1. In this Option, Sue can purchase her car either in 1991 or 1992. Eliminate (A), because it could be true but it also could be false. There are no rules regarding when Ramon purchased his first car, even with our new “if.” If we are looking for what must be true, then, any answer choice that purports to pin down the year in which Ramon bought his car must be wrong. Cross off (B) and (C). In Option 1, Taylor cannot purchase his car in 1994, as that’s the year in which he graduates. Eliminate (D). Taylor must buy his first car in 1995: (E) is correct. Game 4: Alphabet Soup Situation: A child eating alphabet soup Entities: Noodle letters: T, U, W, X, Y, Z Action: The child eats all six letters in three spoonfuls. We’ll have to Sequence the letters as well as Distribute them to the different spoonfuls, so this is a Hybrid Distribution/Sequencing Game.

Page 29: LSAT_PT 50_Expl_web

Limitations: The 6 letters will be eaten in 3 spoonfuls, with at least one and no more than three letters per spoonful. Do the arithmetic: The child will either eat two letters in each of the three spoonfuls, or she’ll eat three letters in one spoonful, two letters in one spoonful, and one letter in another spoonful. The Initial Setup: Our goal for a Hybrid game is to create a sketch that encompasses both actions. Here, we’ll use the spoonfuls as the basis for our sketch—the spoonfuls can be seen as having an inherent order, and they can be seen as the subgroups for our distribution game: T U W X Y Z 1 2 3 The Rules: Rule 1: U is in a later spoonful than T: T . . . U So U cannot go into spoon 1, and T cannot go into spoon 3. Rule 2: U is not in a later spoonful than X. This is a good place to stop and think. This rule does not mean that U always comes before X; this rule makes it entirely possible that U and X could be in the same spoonful of soup. We’ll need to reflect that possibility in our notes: UX or U . . . X Because U cannot go into spoon 1, and X can’t be in a spoon before U, X cannot go into spoon 1 either. Rule 3: Y is in a later spoonful than W: W . . . Y Y cannot be eaten in spoon 1, and W cannot be eaten in spoon 3. Rule 4: U is in the same spoonful as either the Y or the Z, but not both. UY or UZ (You could also write this as U Y/Z ) Never UYZ Deductions: We have the U Y/Z Bloc, which

can only go into spoons 2 or 3, but placing that bloc still leaves much to be determined. Don’t be fooled into thinking that the Numbers make this a Limited Options game. Yes, there are two ways the numbers could break down—2/2/2 or 1/2/3—but that number breakdown doesn’t give us any further information about which spoonful is where, or about which letters are in which spoonful. Not every set of two possibilities is a Limited Options setup, only those which allow us to make two separate sketches and further deductions. We have no Established Entities, either in the rules or by virtue of an accumulation of rules. U is Duplicated in rules 1, 2 and 4, and Y is duplicated in Rules 3 and 4, but despite all of the duplications, there are still too many possibilities to map out. We’ll have to depend on the questions to narrow our possibilities. That’s not too surprising. After all, this is a Hybrid game, so we should expect it to be a little more difficult. The Final Visualization: T U W X Y Z 1 2 3 T . . . U UX or U . . . X W . . . Y UY or UZ ( U Y/Z ) Never UYZ The Questions: 18. (B) Acceptability When one entity must come after another, they cannot be in the same subgroup. Rule 1, T comes before U, is violated in (D). Rule 2, U does not come after X, is violated in (E). Rule 3, W is eaten before Y, is violated in (A) (where Y is placed before W) and (C) (where W and Y are placed in the same spoonful). (B) is the only choice left standing, and is correct.

Page 30: LSAT_PT 50_Expl_web

19. (D) “If” clause / “could be true” Sometimes a new “if” will lead to Limited Options. Our new ”if” for this question puts Y all alone, so we will have a 1-2-3 distribution of entities among the subgroups.. Think a little further: where can Y go? Well, “W…Y” means that Y can’t be in spoon 1, so it must be in either spoon 2 or spoon 3. In order to evaluate what could be true, we’ll have to consider both possibilities. If Y is by itself in spoon 2, W must be in spoon 1, because W comes before Y. T comes before U, so if Y is alone in spoon 2, T must be in spoon 1 and U must be in spoon three. Z must be with U (Rule 4), and so must X (Rule 2), so U, X, and Z must be together in spoon 3. We’ll call this Option 1. If Y is by itself in spoon 3, we aren’t immediately certain where W goes. We can, however, put U into spoon 2 and T into spoon 1 based on Rule 1. Z must be in spoon 2 with U (Rule 4), and because X cannot come before U (Rule 2) and cannot join with Y because of our new “if,” X must join them as well. So U, X, and Z must be in spoon 2, and that spoon is full; W and T remain in spoon 1. Our scratchwork looks like this: Option 1: 1 2 3 W T Y U X Z Option 2: 1 2 3 W T U X Z Y We can now compare our answer choices to our options—the correct answer can be true, and all four wrong answers must be false—and we see that only (D) could be true, in Option 2. (A) Y can never be in the first spoonful because W comes before Y. (B) Because Y is alone, Z must always be with U; U can never be in the first spoonful because T must come before it. (B) must be false. (C) Our options show us that T is always in the first spoonful here. (E) W can never be in the third spoonful because it always comes before Y.

20. (E) “If” clause / “must be true” Be careful not to read too much into a new “if.” This question puts Z into spoon 1. We can start our new sketch based just on that new “if,” then move on to the rest of the rules. The only rule dealing with Z says that the child must eat U with either Z or Y; but because U must be eaten after T, it cannot be in spoon 1. So U is with Y, and they must be eaten in either spoon 2 or spoon 3. We can use that new Bloc of Entities to sketch out Limited Options. T and W must be eaten before U and Y. If U and Y are in spoon 2 (Option 1), then T and W are in spoon 1 with Z and X must be in spoon 3 by itself. If U and Y are in spoon 3 (Option 2), then X must be with them (Rule 2) and T and W can be anywhere in spoons 1 and 2 as long as one of them is in the second spoonful. Here’s our scratchwork: Option 1: 1 2 3 ZW T U Y X Option 2: 1 2 3 Z T/W U Y X T/W We’re looking for a “must be true.” With two sketches, that means we’re looking for something that is true in both of them. Either way we work out this new “if,” X must be in spoon 3, and so the correct answer is (E). 21. (D) Complete (and Accurate) List Sometimes the LSAT writers can ask for a “complete and accurate list” without using those exact words. Here, we are asked to identify the complete list of letters, any one of which could be eaten alone in the first spoonful. In other words, the right answer will be contain all of the letters that could be eaten alone in the first spoonful, and none of the letters that could not be eaten alone in the first spoonful. We can use our prior work to eliminate wrong answer choices. Our scratchwork for question 20 showed us that Z could be eaten all by itself in the first spoonful of soup. That eliminates answer choices (A), (B) and (C).

Page 31: LSAT_PT 50_Expl_web

Now all we need to do is consider whether X can be eaten alone in spoon 1. Rule 2 says that X can’t be eaten before U, and Rule 1 says that U is eaten after T. This means X can’t possibly be eaten in spoon 1, whether alone or with other letters. That eliminates (E) and leaves (D) as the right answer. 22. (A) “If” clause / “could be true” You may be able to answer a question without drawing out all of the new “if’s” possibilities. Our new “if” puts T in spoon 2. Because U must be eaten after T (Rule 1), this new rule puts U into spoon 3. But if U is in spoon 3, then X must be in spoon 3 as well because Rule 2 says that U is not eaten after X. Rule 4 tells us that U must be eaten with Y or Z, but not both. This means that U, X and either Y or Z must be in spoon 3, and we have enough information to answer the question. There are three letters in spoon 3, so (D) and (E) must be false. Only one spoon can hold 3 letters, so (B) and (C) also must be false. That leaves only (A) as the correct answer choice, and indeed, we could have two letters in the first spoonful: 1 2 3 WZ T U X Y

Page 32: LSAT_PT 50_Expl_web

SECTION D LOGICAL REASONING 1. (E) Inference If a prediction for an Inference question pops into your head, feel free to scan for it in the choices. Most of the time, it’s not a good idea to try to predict an answer to an Inference question. There are just so many things that would have to be true based on the stimulus that it’s difficult to pick the right one on the first try. So predicting an answer to Inference questions is usually a waste of time. Of course, sometimes an obvious Inference will jump out at you; if that happens, it’s fine to look for that in the choices. The first question in this section gives us a simple experiment: two groups were given the same article with different headlines. As we might expect, the groups came up with different answers when asked what the article was about, although the answers within each group were similar. If this led you to think that the headlines must have somehow affected the groups’ answers, you weren’t alone. Let’s use that to scan the choices: (A) is a classic example of an Extreme choice, and one that’s easy to fall for with our focus on the headlines. Notice that the stimulus never said that the readers’ impressions didn’t have to be based on the headlines alone. Eliminate. (B) Nothing in the stimulus suggests that the headlines hampered the readers’ comprehension of the articles; perhaps the headlines were both accurate, but emphasized different aspects of the article. Eliminate. (C) The only students mentioned in the stimulus are first-year undergraduates, so we don’t have any basis for comparing them to more senior students. Eliminate. (D) Headlines in general aren’t necessarily misleading; this is another Extreme choice. Eliminate. (E) hits it right on the head. The only difference between the two groups was the headlines, so they must have been the reason for the readers’ different interpretations of the article. That means they must have influenced the readers’ interpretations. (E) is the correct answer. 2. (C) Flaw (Formal Logic)

Certain classic flaws appear frequently on the LSAT. The “All X are Y” formulation in the first sentence should tip you off that you’re dealing with Formal Logic. “All works of art are beautiful and have something to teach us” is the same as “If something is a work of art, then it is beautiful and it has something to teach us.” We can quickly form the contrapositive of this statement: “If something doesn’t have anything to teach us, or if it’s not beautiful, then it’s not a work of art.” The conclusion uses the same terms as the original Formal Logic statement in the evidence, but it gets them backwards—if something is beautiful and instructive, then it must be a work of art. Well, being beautiful and instructive are necessary characteristics of works of art, but they aren’t by themselves sufficient; maybe works of art also have to be man-made, or maybe they have to be in a museum. Regardless, this argument confuses necessity with sufficiency, one of the most common flaws on the LSAT. We find it re-phrased in (C). (A) Vague terms in an argument are fine, as long as the author doesn’t try to use them to mean different things at different times. He doesn’t do so here, so this isn’t a flaw. (B) “Evaluative conclusion?” “Claims about factual matters?” Sounds like the LSAT is trying to get us to pick an answer choice because we’re not sure what it means, which is never a good idea. (D) The argument never tries to relate “beautiful” and “instructive” to each other; it only deals with things that have both of these qualities. (E) Actually, the stimulus mentions things that are both beautiful and instructive, but not part of the natural world: works of art. The argument can’t fail to consider things that it talks about. 3. (B) Principle A Principle is a law-like rule that you can use to explain the situation at hand. Stories about research breakthroughs always seem to involve some kind of happy accident like the ones in the stimulus. Maybe groundbreaking events in science always occur through some lucky chance; or maybe nobody remembers the boring stories about the researcher who finally got to his new theory after years of drudgery in a lab. Either way, the stimulus gives us two examples of breakthroughs that came from a shift in focus, and says that history tells us

Page 33: LSAT_PT 50_Expl_web

about many more. All of those situations would follow a simple rule: a shift in focus can lead to important breakthroughs, (B). (A) The stimulus doesn’t actually suggest that the breakthroughs were the result of chance, just a shift in thinking. Maybe Copernicus and Jenner consciously tried to look at their problems in different ways. (C) The stimulus only touches on the results of seeing things in a new light; the ability to do so is outside the scope. (D) Breakthroughs that came from the accumulation of new information are outside the scope of the argument, so we can’t compare them with breakthroughs stemming from better organization of available information. (E) The amount of information available in the fields mentioned in the stimulus is likewise beyond the scope of the argument. 4. (E) Flaw An argument’s flaw is often closely related to the author’s Assumption. This politician believes that censorship is not morally wrong, and tries to get us to agree. He does so by presenting us with an analogy, and arguing that it would be absurd to say that this analogous situation was also morally wrong. But his analogy doesn’t really make much sense—an actor refusing a role in a film doesn’t sound like censorship. The politician, though, certainly seems to think that they’re one and the same. That leads us right to his flawed assumption: that declining a film role is, in a way, censorship. (E) points out this flaw. (A) Everything you hear about Hollywood might lead you to believe that actors are liberal, but this argument doesn’t make the connection. (A) is outside the scope. (B) is tempting, particularly given all the name-calling in politics. But this politician’s argument is really about censorship itself, and not the views of liberals. (C) is far too broad, since the only profession mentioned is acting. (D) The politician’s conclusion isn’t inconsistent with his argument, even though neither one of them makes much sense. 5. (E) Weaken

Be on the lookout for alternative possibilities in Weaken questions. Just when you think you know something about how to take care of your car, a study like this always comes along to tell you differently. According to this author, cheap brands of motor oil are just as good as the expensive brands when it comes to retarding wear on pistons and cylinders. But does that really mean that the cheap brands are “the best buys?” Well, sure…if all they have to do is retard engine wear. If motor oil serves some other function, as (E) suggests, maybe the cheap brands aren’t so great. (A) actually strengthens the argument, by suggesting that people who know a lot about cars choose the cheaper brands. Eliminate this 180. (B) Tests other than the one in the stimulus are outside the scope of the argument. (C) Even if (C) is true, how would it affect the argument? It doesn’t tell us anything about how the cheaper motor oil relates to the more expensive brands. (D) The “certain brand of oil” in (D) might be a cheap brand, or it might be an expensive one. Without more information, we can’t say how this choice would affect the argument. 6. (C) Method of Argument Don’t skip to the second argument in a dialogue; it’s often impossible to understand until you’ve read the first one. Elena’s conclusion is pretty simple: democracy is the best form of government. She gets there by asserting that the best form of government is one that fosters its citizens’ belief that they have a say in how their government is run. It would be a good exercise at this point to find Elena’s assumption: a democracy is the only government in which citizens believe that they have a say in how the government is run. Marsha quickly points out the flaw in this argument: there are other forms of government in which citizens believe they have a say in how the government is run, even though that isn’t true. Elena’s evidence could also be used to support the conclusion that these governments are the best form of government. That would contradict her conclusion, as (C) points out. (A) The word “but” at the opening of Marsha’s statement should tell you that she disagrees with Elena, making (A) a 180.

Page 34: LSAT_PT 50_Expl_web

(B) and (D) say virtually the same thing, and it’s off the mark in both cases. Marsha’s unstated conclusion is that Elena’s argument doesn’t make much sense. (E) Marsha doesn’t necessarily reject Elena’s conclusion, she just takes issue with how Elena got there. 7. (B) Flaw Flaw questions will often ask you to think of an unexamined alternative, just like Weaken questions. It seems like every new drug you hear about is the best thing to come along in years. Of course, after some of them have been on the market for a while, you start hearing about all the nasty side effects, and finally there are calls to pull it off the shelves. The researcher has skipped straight to the last stage of this process. She points out all of the nasty side effects caused by the newest drug for a certain disease, and argues that its use should be discontinued. Unfortunately, she doesn’t say anything about all of the other drugs that are used to treat the disease—maybe they’re equally bad, or even worse. Or maybe the disease itself is worse than the side effects from this drug. In any case, there could be reasons we shouldn’t stop using this new drug, all of which the researcher ignores. (B) points out this flaw. (A) The argument doesn’t need to specify what is meant by a term that has an obvious meaning. (C) isn’t necessarily true—the researcher only argues that we should stop using the new drug, regardless of what other drugs patients might be using. (D) is true, since the author doesn’t consider this factor, but that isn’t a flaw; she doesn’t consider it because it doesn’t affect her argument. (E) the researcher actually does acknowledge that the effects of the treatment may not be the same in all cases, by saying that the treatment “usually” has severe side effects. 8. (B) Point at Issue Use the Kaplan Decision Tree to quickly answer Point at Issue questions. Of all the things you would expect to consider in life, Aristotle’s principle of justice as it applies to the scheduling of dental appointments was probably

pretty low on the list. Nonetheless, we’ll consider it now. Otis believes that Aristotle’s principle would preclude dentists from scheduling an appointment to benefit a friend, but Tyra thinks that human nature trumps Aristotle’s principle of justice in this case. With any Point at Issue question, you can evaluate the choices by asking three questions about each one: Does the first speaker have an opinion on this? Does the second speaker? Do their opinions differ? If you can answer “yes” to all three questions, you’ve found the correct answer; otherwise, you should eliminate the choice. Let’s do that here: (A) Both speakers have opinions about Aristotle’s principle of justice, but they would probably agree on this choice. Eliminate it. (B) may sound too abstract to consider, but look closer: Otis argues that Aristotle’s principle suggests that we should treat friends and others as relevantly similar cases, while Tyra argues that the cases are different enough to warrant using different rules. The two speakers disagree over (B), which makes it the correct answer. For the record: (C) The two speakers would each have an opinion on this choice, but they’d probably agree. (C) basically says that it’s impossible to follow Aristotle’s principle of justice, but both speakers seem to think that’s not true. Eliminate this choice. (D) Again, both speakers would probably agree that (D) is not the case. Eliminate. (E) Neither speaker ever discusses whether Aristotle’s principle recognized an exception for friendship. Eliminate. 9. (A) Strengthen Read the question stem carefully to be sure you correctly identify the question type. This is actually a Strengthen question, since we’re asked to justify the author’s reasoning, but you might have been thrown off if you stopped at the word “assumed” in the stem. Thankfully, it shouldn’t have given you too much trouble, since your analysis of a Strengthen stimulus should always include finding the author’s assumption. One of the few pieces of health advice that’s been consistent over the years is that fat is bad for your heart. Of course, even that advice has to be qualified: saturated fat is much worse for your heart

Page 35: LSAT_PT 50_Expl_web

than unsaturated fat. This means that, as this argument tells us, people who replace saturated fat in their diet with unsaturated fat will decrease their risk of heart disease. So people who eat a lot of saturated fat can just increase their intake of unsaturated fat and lower their risk of heart disease, right? Not so fast. Increasing your intake of unsaturated fat isn’t necessarily the same thing as replacing saturated fat with unsaturated fat. But the author equates the two, which must mean that he’s assumed you can’t do one without doing the other, as (A) says. (B) Benefits other than a reduced risk of heart disease are outside the scope of the argument. (C) Diet doesn’t have to be the most important factor in a person’s risk of heart disease; as long as it’s a factor, the argument will still hold up. (D) Life expectancy is outside the scope of the argument. (E) Perhaps this is true, but the argument doesn’t discuss cutting overall fat intake—it just talks about replacing one kind of fat with another. 10. (E) Inference (Formal Logic) Always translate and combine Formal Logic statements in Inference questions. Mediation is increasingly used as an alternative to litigation, and this author tells us a few things about when each option is appropriate. Only those people who are willing to compromise should use mediation, and only those who are sure they are correct should use litigation. By changing “only” into “then,” we can turn each of these assertions into Formal Logic statements: If you want to undergo mediation, then you must be willing to compromise; and if you want to pursue litigation, you should be sure that you are correct. You should also form the contrapositive of each statement, since it’s often crucial to finding the correct answer: If you aren’t willing to compromise, then you shouldn’t undergo mediation; and if you aren’t sure that you’re correct, then you shouldn’t pursue litigation. The final statement gives us a new fact, which can also be turned into an if/then: If your conflict is based on ideology, then you aren’t willing to compromise. A quick glance over our translations allows us to combine the final statement with the rule on mediation. If your conflict is based on ideology, then you shouldn’t undergo mediation to resolve your conflicts. We can use this new fact to evaluate the answer choices.

(A) People who do not undergo mediation might need to be sure of their positions, or they might not. We can’t be sure from the stimulus. Eliminate. (B) People whose conflicts are not based on ideology are outside the scope of the argument. Eliminate. (C) tries to combine the statements about ideology-based conflict and litigation, but the terms of those statements don’t allow us to make any such link. Eliminate. (D) Being sure of the correctness of your position means that you’ve fulfilled the necessary condition to pursue litigation, but the stimulus never says that this means you shouldn’t undergo mediation. Eliminate. (E) matches the combination of the Formal Logic statements perfectly. Isn’t it wonderful when that happens? This is the correct answer. 11. (D) Assumption The assumption is a statement that must be true in order for the conclusion to be true. Once again, we get a bit of health advice from the LSAT. According to the author, scientists’ long-standing suspicion, that a diet that contains some fish provides health benefits, has new support. A recent study provides that support. In the study, middle-aged people who ate fish twice a week were much less likely to develop heart disease than those who did not eat fish. This study may sound like it supports the scientists’ ideas, but we know from the fact that we’re looking for an assumption that there must be a missing piece. Well, what if there was some other reason why the fish-eating group was less likely to develop heart disease? That would tear the author’s argument apart. So he must be assuming that there is no other reason. In fact, this general assumption—‘there’s no other way this could have happened’—is quite common on the LSAT, and we find it expressed in (D). (A) and (C) Vegetarians are way outside the scope of this argument, as is red meat. (B) was probably very tempting, since the wording seems to link something about the fish-eating group to heart disease. But if you look closely, (B) suggests that the fish-eating group would actually have more heart disease, contradicting the author’s argument.

Page 36: LSAT_PT 50_Expl_web

(E) “Sedentary occupations” aren’t so closely linked to heart disease that eliminating their influence is necessary to the argument. 12. (A) Weaken Irrelevant comparisons are often tempting wrong answer choices for Weaken questions. By now, you should have learned to distrust the results of any study on the LSAT. In fact, we wouldn’t be surprised if you start distrusting the results of real studies, since the studies you’ve been working with have so many flaws. The studies that this researcher points to are no exception. They point to similar levels of improvement in short-term psychotherapy clients, regardless of the kind of psychotherapy they use. The author takes a rather large leap from these studies, concluding that the improvement must be the result of something common to all psychotherapies. There are quite a few assumptions necessary to bridge the gap between the studies and the researchers’ conclusion about them, not least that the studies showed actual improvement. (A) knocks this assumption out of consideration, suggesting that the improvement in the studies wasn’t really worth mentioning. (B) Comparisons between short-term and long-term therapy are irrelevant to the argument, which only deals with short-term psychotherapy. (C) Comparing trained counselors and untrained laymen is likewise irrelevant to the argument. (D) Even if the specific techniques used by different therapists differ dramatically, there could still be common elements in all types of psychotherapy. (E) is yet another irrelevant comparison. The training levels of the psychotherapists and the range of techniques they use are irrelevant to the argument. 13. (B) Assumption Beware of Assumption answer choices that simply restate part of the stimulus. Historical photographs are sometimes used in articles about current events. The author argues that this practice overstates the similarities between the past and the present. That may not sound like such a bad thing, but that’s not where the author stops. He claims that the practice keeps us from forming a good understanding of the present, since it presents current events as mere repetitions of past events.

Hang on, you might be saying. Just because there are photographs of past events along with a story doesn’t mean that everyone will think history is repeating itself. After all, don’t those pictures usually come with captions that at least give some context to help us distinguish them? This author doesn’t have a very high opinion of the public—he assumes that the public’s understanding of current events will be distorted by the use of these historical photos. (B) puts together all the pieces of this assumption. (A) and (C) The assumption is an unstated but necessary connection within the author’s argument, but these choices simply restate connections that are already made in the text. (D) The author doesn’t need to assume that there’s no other way to do what he warns against; he only argues that this particular practice does so. (E) Journalists’ beliefs are outside the scope of the argument. 14. (D) Parallel Reasoning (Formal Logic) You can use Formal Logic to put an argument into abstract terms, which will help you compare it to the answer choices. If we get rid of all of the specifics of this argument, it’s really quite simple: If A happened, then B probably wouldn’t have happened. But we know that B happened, so A probably didn’t happen. This is really nothing more than an if/then statement and its contrapositive, even with the “highly unlikely” thrown in. We can use an abstract formula to find the correct answer choice: “If A probably not B; B, so probably not A.” (A) There’s absolutely no link between the two statements in (A); an equal chance of winning the tournament doesn’t have anything to do with a soccer team losing its goalie. Eliminate. (B) is close, but it follows a slightly different logical pattern. This choice says, in effect, if someone sells insurance, they’re probably friendly, and Kapinski sells insurance, so he’s probably friendly. This is just, “If A, then probably B,” without a contrapositive. Eliminate. (C) Instead of asserting that something was highly unlikely, (C) asserts that something was likely, turning the conclusion of the argument on its head. Eliminate. (D) gives us a perfect parallel. Use “Clarissa missed her bus” as A and “Clarissa getting to work on time”

Page 37: LSAT_PT 50_Expl_web

as B, and the logic works exactly the same. (D) is the correct answer. Eliminate. (E) The conclusion in this choice lacks the “probably” language in the stimulus, so it cannot be correct. 15. (D) Inference (Formal Logic) Don’t let the LSAT confuse you with abstract terms. Sonya has a rather bleak outlook on life. According to her, if you live without constant awareness of the fragility and precariousness of human life (Yikes!), then your mind is clouded by illusion. But if you are constantly aware of this fragility and precariousness, then you’ll taint your emotional outlook on life. Notice that each of these statements can be phrased as if/thens, and that they share “the constant awareness of the fragility and precariousness of human life.” If we form the contrapositive of either one of these statements, we’ll find that we can link the two as surely as we would, “If A, then B; if B, then C.” Let’s start from the second statement. Its contrapositive would be, “If someone’s emotional outlook on life is not tainted, then they aren’t constantly aware of the fragility and precariousness of human life.” Well, we’d know from the first statement that people who aren’t aware of the fragility and precariousness of human life have minds that are clouded by illusion. As abstract as they are, we can combine these statements just like any other Formal Logic statements: “If someone’s emotional outlook on life is untainted, then their mind is clouded by illusion.” Form the contrapositive of this statement, too: “If someone’s mind isn’t clouded by illusion, then their emotional outlook on life is tainted.” Cheery, isn’t it? Well, let’s use these statements to evaluate the answer choices anyway: (A) People’s priorities are outside the scope of the argument. Eliminate. (B) Sonya doesn’t set up an all-or-nothing proposition for the entire world, just for each individual. (B) is thus too Extreme. (C) “Self-deception” sounds like clouding your mind with illusion, but Sonya never says it’s impossible to live without doing this; in fact, she gives us the alternative to it, tainting your emotional outlook on life. Eliminate. (D) perfectly combines the logical statements in Sonya’s argument. If you did so before you went

through the choices, this question probably wasn’t so bad. (D) is the correct answer. For the record: (E) Sonya never makes a value judgment about either proposition. In fact, she seems to think they’re both equally awful. Eliminate this last option. 16. (C) Paradox The correct answer to a Paradox question must reconcile the two sides of the paradox. Most of us know that we should never go grocery shopping when we’re hungry, since we always end up buying more than we really need. But it turns out that we should always go shopping with a list, as well, and avoid things that are on sale. According to this stimulus, those who shopping without a list spent more money than those with a list, even though the shoppers without lists only bought items that were on sale, and the shoppers with lists stayed away from the sale items. That certainly seems paradoxical, since we’d expect that sale items would be cheaper than items at full price. But (C) gives us an excellent explanation—it turns out that the sale items were already the most expensive items in the store, so even on sale they could have cost more than other items at full price. (A) Shopping carts are outside the scope of the argument, since they shouldn’t affect how much money the shoppers spent. (B) sounds good at first—if the shoppers without lists bought unnecessary items, they’d spend more, right? Well, not necessarily. Even unnecessary items can be cheap, and (B) still doesn’t explain how the listless shoppers bought only sale items but still spent more money. (D) touches on the cost of items at the grocery store, but it doesn’t tell us which group bought more of the expensive items, so there’s no way of comparing the two sides of the paradox. (E) If anything, (E) deepens the paradox. If there were only a few items on sale, it’d be even harder to explain how the shoppers without lists spent more money buying only sale items. 17. (A) Flaw Be sure to get all the information you can out of the question stem.

Page 38: LSAT_PT 50_Expl_web

This question uses a particular type of classic flaw, and we can tell that before we even look at the stimulus. Whenever the question stem tells us that an argument is flawed because it “overlooks a possibility,” we know we’re looking for an unexamined alternative. Sir Edmund Hillary said he climbed Mount Everest “because it was there,” but this stimulus gives us a reason to avoid imitating him. Once the climbers mentioned here got past a certain height, they experienced all sorts of cognitive difficulties, including slurred speech. The author seems to think that the fact that all of these difficulties occurred together means that the area of the brain controlling speech can’t be distinguished from other areas of the brain. But we know from the stem that he’s overlooking some possibility, and if we think a bit about his conclusion it shouldn’t be too hard to figure out. What if all of the climbers’ cognitive difficulties came about because oxygen deprivation affects the entire brain, as (A) says? Then the part of the brain that controls speech could be distinct from the areas of the brain controlling other functions, but the climbers would still show speech difficulties along with other problems. (B) makes an irrelevant comparison between the climbers’ capabilities before the study and some average capability level, but we only care about their capabilities during the study. (C) Different levels of impairment in different types of cognition wouldn’t necessarily mean that these types of cognition arose from different areas of the brain—this possibility is irrelevant to the author’s argument. (D) The author’s argument draws a conclusion about the brain based on the climbers’ difficulties, but that conclusion wouldn’t be changed by when their difficulties appeared. (E) The climbers’ training is outside the scope of the argument, especially since it didn’t seem to have much effect on their oxygen deprivation. 18. (D) Principle Keep a close eye on the scope of the author’s conclusion. There’s a long history of pesticides being banned due to their harmful environmental effects, as this stimulus points out. But here we’ve got a curious problem arising from the status of three different pesticides. TSX-400 was thought to be extremely harmful, and was banned as a result. The two other pesticides, Envirochem and Zanar, were thought to

be harmless, so they’re still in use. The problem is that recent studies have shown Envirochem and Zanar to be even more harmful than TSX-400. We might suspect that on the basis of this information, the author would suggest banning the two pesticides currently in use, but that’s not quite what we get. He actually argues that we should ban Envirochem and Zanar or we should legalize TSX-400. That “or” is what will lead us to the right answer. The author doesn’t argue that any harmful pesticide should be banned, he just wants consistency in the law—if certain pesticides cause similar levels of environmental harm, we should either ban or legalize all of them. (D) most accurately expresses this principle. (A) and (C) don’t quite match the stimulus, since they only discuss when pesticides should be banned. (B) doesn’t match the stimulus either, because it would advocate banning any harmful pesticide. (E) also suggests that legal pesticides should be harmless, while the author seems to think a harmful pesticide can be legal as long as any harmful pesticide is legal. 19. (A) Flaw Be wary of arguments that involve correlations. Smoking is correlated with all sorts of nasty results, so it should be no surprise that scientists have found a correlation between smoking and heart disease. But the surprise in the stimulus is the correlation between smoking and drinking caffeinated beverages. Even more surprising is the conclusion that the author draws from these correlations: a third correlation exists between drinking caffeinated beverages and developing heart disease. Whether or not you have any experience with statistics, experience with flaw questions on the LSAT will soon teach you that it’s almost impossible to draw logical conclusions from correlations. Evidence like this is particularly suspect. Two correlations drawn from a single group shouldn’t be used to draw a conclusion about a third correlation, because they don’t necessarily overlap. (A) points out that the author overlooks exactly this possibility: maybe the smokers who drink lots of caffeinated beverages aren’t the same smokers who have a higher risk of heart disease. (B) Other factors in the development of heart disease are outside the scope of the argument.

Page 39: LSAT_PT 50_Expl_web

(C) and (D) If these choices were true, they would actually support the author’s argument that drinking caffeinated beverages and developing heart disease are correlated. (E) would create an argument with an entirely different logical flaw, by turning a mere correlation into cause-and-effect. Logically, this is a no-no. 20. (E) Principle With Principle questions, you can look for a 1:1 matchup between the stimulus and the correct answer. Just when we thought it was safe to walk into a supermarket, the LSAT tells us that every time we walk though the doors we’re subject to a clever ploy that forces us to walk past tempting marketing displays just to buy a loaf of bread. Of course, grocery stores’ ingenuity in crafting the layout of their stores doesn’t just lead to more sales; it also leads to customer complaints about inconvenience. We can sum up this argument in a relatively simple principle: forcing people to take certain actions may have unintended consequences. (E) matches every part of this proposition. (A) A recommendation about which customers supermarkets should focus on are outside the scope of the argument. (B) is too Extreme—customers may complain about the inconvenience of shopping in supermarkets, but there’s no indication in the stimulus that it leads them to shop elsewhere. (C) is likewise Extreme. There’s no indication in the stimulus that the supermarkets’ devious marketing ploy has failed. (D) If distracting customers with displays of tempting items isn’t good for business, why would every supermarket do it? (D) seems to contradict the stimulus. 21. (C) Assumption The author’s central assumption is an implicit connection between the different parts of their argument. This section of the LSAT certainly has had a lot of health tips, although this is the first one to come from a doctor. Apparently anyone who wants to reduce their blood pressure should rely more on lifestyle changes than medication, since the doctor’s orders tell us that lifestyle changes are healthier. But

let’s look a little closer at his evidence for that proposition. Blood pressure medication often has unhealthy side effects, but lifestyle changes will reduce blood pressure just as effectively as the medication. If the lifestyle changes are healthier, then they can’t have any similar unhealthy side effects. But the doctor never comes right out and makes this link explicitly. He just assumes that it is true, as (C) points out. (A) There could be other ways to reduce blood pressure, but they are outside the scope of the doctor’s argument. (B) The stimulus already tells us that lifestyle changes are as effective as medication at reducing blood pressure. This doesn’t give us the reason why lifestyle changes are healthier. (D) is far too broad, since it encompasses all medical conditions, not just blood pressure. (E) distorts the stimulus. According to the doctor, the two methods of treating high blood pressure have similar effectiveness, but he suggests they have different side effects. 22. (E) Flaw Watch out for arguments that confuse numbers with proportions. Beware of crossing the street, this argument seems to say, but if you must, be sure to jaywalk. That’s the implication of the conclusion that it’s less dangerous to jaywalk than it is to cross a street with the light. But think closely about what that means. If it really were more dangerous to cross with the light, that would mean your chances of being killed would be greater when you cross with the light than against it. But that’s not what the evidence establishes. The evidence only tells us that more pedestrians are killed crossing with the light than against it. This could be because it’s more dangerous to cross with the light…or it could be because far more people cross with the light than against it. (E) points out the latter possibility. (A) Nothing in the argument suggests that either the studies or the statistics in the evidence were biased. (B) and (C) There is no causal relationship presented in the argument, whether between two correlated events or between them and a common cause.

Page 40: LSAT_PT 50_Expl_web

(D) The evidence and the conclusion deal with the same environment—North American cities. There is no need to consider other environments. 23. (C) Strengthen To Strengthen an argument, try to make it more likely that the conclusion will follow from the evidence. The LSAT may have given you a few sleepless nights—at least, before you started studying with Kaplan—but that’s nothing compared to true insomnia. Unfortunately, this stimulus suggests that melatonin won’t help with true insomnia. Even though melatonin tablets can induce sleep, the stimulus tells us that melatonin didn’t always significantly induce sleep in studies, even among people with normal sleep patterns. But most of the studies didn’t involve people with insomnia, so there’s a gap in the author’s argument. We’ll have to connect the results of those studies to people with insomnia. (D) does so pretty well, by telling us that only people without insomnia were affected by melatonin, even in the studies that included people with insomnia. (A) You might initially think that the weaker correlation in experiments that included subjects with insomnia argues that melatonin isn’t as effective on people with insomnia, but (A) doesn’t exclude the possibility that some other aspect of these studies accounted for the weaker correlation, so it might not affect the conclusion at all. (B) would Weaken any conclusion drawn from the studies in the stimulus. (D) This choice deals with the placebo effect—it doesn’t tell us anything about the effects of melatonin. (E) goes way too far, suggesting that melatonin would be 100% effective for insomnia sufferers. But think about it—even if this were the case, the few subjects affected by melatonin in the stimulus could all have been people with insomnia. If that were the case, (E) would actually weaken the argument. 24. (D) Parallel Reasoning (Flaw) Try to characterize the flaw in Parallel Flaw questions; it is often easier to use in evaluating the answer choices. The substance of the stimulus in a Parallel Reasoning question is never important. The logical process is always where your focus should be. Here,

we shouldn’t worry too much about the details of how the triceratops went extinct. Instead, we’ll look at how the author’s argument is flawed. Conclusive proof is always difficult to come across. But this author takes the desire for conclusive proof to the extreme. The initial event in the stimulus caused two results—the asteroid caused a firestorm and climate change. We can’t prove that either of two results caused a later event (that either the firestorm or the climate changes cause the triceratops to go extinct). So, according to the author, we can’t say that the initial event, which caused both results, was really responsible for the final event—that the asteroid impact caused the triceratops’s extinction. Well, perhaps we can’t prove that the asteroid impact caused the extinction, but it certainly sounds like the extinction could be caused by the impact itself, or by the combination of the two results. The fact that we can’t conclusively prove one link of a causal chain doesn’t mean that the chain breaks down altogether—this is the flaw in the author’s argument. We’ll find a similar causal chain in (D). (A) There’s no initial event in this chain of causation, only Leon and Pam. While they might be analogous to the firestorm and the climatic changes, there’s nothing we can analogize to the asteroid. (B) and (E) give us the either/or distinction that was missing in the stimulus: our only options are A and B, so we’ll end up with either A or B. (B) makes a slight mistake in its logic (we might not know who will win the election) and (E) does not, but neither one matches the stimulus. (C) sounds very similar to the stimulus, but again, there’s no initial event that caused John’s erratic driving. Perhaps if the stimulus had started off by saying, ‘John’s recklessness caused him to drive at excessive speeds and weave out of his lane,’ it would be parallel to the stimulus, but absent that initial cause we must eliminate this choice. 25. (E) Main Point Don’t get bogged down in the details when a question asks you for the author’s Main Point. When looking for the author’s conclusion, it’s always helpful to use the One Sentence Test. Here’s how it works: try to cut everything from the argument except for the one sentence that the author wouldn’t be willing to part with. That single sentence will be their conclusion.

Page 41: LSAT_PT 50_Expl_web

Don’t be fooled by the sheer length of the economist’s argument. His point is actually quite simple, and it shows up early on. In fact, we could cut all of the evidence that finishes up the stimulus, and just stick with the gist of the first sentence: across-the-board cuts for all tax brackets tend to disproportionately benefit the wealthy. This is restated in (E). (A) This proposition is within the same sentence as the economist’s main conclusion, but the Keyword “Although” at the opening of the sentence should tell you that it’s a subsidiary point. (B) is a subsidiary detail that leads to a lesser point, that decreasing income taxes across the board leads to either an increase in taxes that disproportionately hurt the poor or an increase in borrowing that disproportionately benefits the wealthy. (C) points to a detail that arises in the middle of the stimulus. (D) is yet more evidence that the economist uses to prove his conclusion.